You are on page 1of 61

Prime Mock 15 (CLAT) 2024

English
Directions for questions 1 to 30: Read the passages given below and answer the questions that follow.

Passage – 1

Some experts term the third-gen web as 'Web3', while others use the descriptor 'Web 3.0'. For web-tech buffs, Web3 is decentralised, privacy-
oriented, blockchain-driven and crypto-asset friendly; while Web 3.0 upholds the property of the 'semantic web,' which is powered by Artificial
Intelligence (AI). The real point about the semantic web is its ability to recombine information available on different websites to generate new content
and knowledge resources that are more authentic and creative. Followers of Web 3.0 claim that their version is endowed with robust capability on
the data analytics front. This way, it is argued that Web 3.0 will create far better search engines.

Of the two variants, Web3 seeks to radically transform the manner in which data is generated, monetised, shared and circulated. Further, it
advocates decentralised data storage systems with the objective of unshackling the oligopolistic grip of technology behemoths over data. Web3 has
file-sharing systems such as the Inter-Planetary File System which are cryptographically protected, more secure and capable of functioning off
Internet and off blockchains. In this manner, Web3 seeks to overcome the data storage barriers of blockchains.

Web3's boldest element is the strategic role it assigns to non- custodial wallets that function as digital passports for users to access blockchain-
enabled transaction platforms. These wallets aid the creation of a 'ownership economy,' whereby creators themselves control their content.
Fundamentally, they work as the digital proof of identity.

Web3 seeks to replace micro-economic organisations with decentralised autonomous organisations (DAOs). At a more macro level, it seeks to
create a distributed economic system, where special classes of native digital tokens and cryptocurrencies would form the media of monetary
circulation. In general, Web3 platforms would serve to raise the efficiency of peer-to-peer transactions.

Apart from utility tokens that enable users to access life support services, Web3 systems also seek to generate fungible digital assets to reward local
providers of data storage capacity for their services. Asset tokens that are native to the new-gen web have the potential to function as capital
mobilisation tools for Web3 projects. Likewise, stakeholders of DAOs can utilise tokens to exercise their voting rights. The NFTs of Web3 are more
dynamic as they seek to incorporate improvements brought in by incremental innovations.
Q 1. 30630558  What is the main idea of the passage regarding the third-generation web and its variants?

a)  Web3 aims to transform data generation, monetization, and storage systems while advocating for decentralization and empowering creators.

b)  
Web 3.0 upholds the concept of the semantic web, which combines information from different websites using AI to generate new and authentic
content.
c)  Web3 and Web 3.0 both prioritize data analytics capabilities and aim to create more efficient search engines.
d)  The strategic role of non-custodial wallets in Web3 is to provide secure digital identities and enable the creation of a distributed economic system.

Q 2. 30630558  Based on the last paragraph of the passage, what can be inferred about the role of native digital tokens and cryptocurrencies in
Web3?

a)  Native digital tokens and cryptocurrencies are primarily used for data storage purposes in Web3.

b)  Native digital tokens and cryptocurrencies are exclusively used for capital mobilization in Web3 projects.
c)  
Native digital tokens and cryptocurrencies serve various purposes, including rewarding data storage providers, exercising voting rights, and
facilitating peer-to-peer transactions in Web3.
d)  Native digital tokens and cryptocurrencies have limited functionality and are not integral to the functioning of Web3 platforms.

Q 3. 30630558  According to the 4th paragraph of the passage, which statement accurately describes the strategic role of non-custodial wallets in
Web3?
a)  Non-custodial wallets provide secure digital identities and facilitate the creation of a decentralized economic system in Web3.

b)  Non-custodial wallets primarily function as file-sharing systems for encrypted data storage in Web3.
c)  Non-custodial wallets enable the use of utility tokens for accessing life support services in Web3.

d)  Non-custodial wallets serve as digital proof of identity and facilitate the exchange of native digital tokens in Web3 projects.

Q 4. 30630558  Web3 seeks to replace micro-economic organizations with ____________ (DAOs), and aims to create a distributed economic
system where special classes of ____________ and cryptocurrencies would form the media of monetary circulation.

a)  Microchips; blockchain
b)  Data storage; utility tokens

c)  Decentralized autonomous organizations (DAOs); peer-to-peer networks

d)  Decentralized autonomous organizations (DAOs); native digital tokens and cryptocurrencies

Q 5. 30630558  What does the term "fungible" mean in the context of Web3 platforms?

a)  The ability to generate new content and knowledge resources using AI in Web3.

b)  The cryptographic protection and security of file-sharing systems in Web3.


c)  The capability of non-custodial wallets to provide digital proof of identity in Web3.

d)  The interchangeability and identical value of digital assets or tokens in Web3.

Directions for questions 1 to 30: Read the passages given below and answer the questions that follow.

Passage – 2

I sat down to write a column on the breakneck nature of our thoughts and actions. Using the convenient excuse of a hurried lifestyle and a zillion
things that need to get done immediately, we have reduced our lives to a series of Instagram stories. But before I could type even one word, my
Twitter feed informed me that the National Council of Educational Research and Training has, among other things, decided to drastically reduce the
content on the Mughal empire as well as erase references to the 2002 Gujarat riots and the Government of India's ban on the Rashtriya
Swayamsevak Sangh after the assassination of M.K. Gandhi from its textbooks. Though these two subjects seem disparate, they are indeed
connected.

The forced reduction of time, space and place is a form of violence; an oppressive act perpetrated by a powerful group on the rest of society. They
convince us through multivarious methods, such as coercion, inducement, advertisement and the removal of other choices, that limited
comprehension and expression is the best way forward. Whether it is word, song, movement or speech, we are forced to comply with this new social
regulation. There is another surreptitious act these dominants indulge in. They parallelly choreograph the content that we interact with. We are not
even aware that our minds and hearts are being bent towards a pre-ordained direction and, consequently, weaned away from many invaluable
aspects of knowledge. After a point of time, even if someone points to that light at the end of our normalised dark tunnel, we do not want to go
towards it. We have bought into dingy, narrow darkness as the complete truth and hate whatever lies in the light.

The erasures and manipulations in our textbooks are similar acts of oppression. They intend to not only teach a skewed version of history but also
instil anger and hatred by acts of omission and commission. We have not as yet seen the fine print of the changes implemented by the NCERT but
the overarching intention is the flattening of history. Detailing and interpretations may have been replaced with stony words that force only one kind
of understanding. This form of tyranny is not merely intentional a-historicity; it instils dislike of certain groups of people and the adulation of another
set of people. It narrows the minds of the young and their ability to absorb complexity and nuance remains undeveloped.
Q 6. 30630558  Which of the following according to the author is an act of violence?

a)  Forceful limiting of our comprehensiveness and our expressiveness.


b)  Taking refuge in social media networks like Instagram.

c)  Reducing our lives in the fashion of social media stories.

d)  Finding oneself in opposition to a powerful group which controls society.

Q 7. 30630558  Which of the following would the author agree to be the essential experience of those living in an oppressive society?

a)  Offering a broad point of view of life.

b)  Fearing the consequences of failure in life.


c)  Shunning light in favour of gloom.

d)  Refusing to interact with content governed by the societal elites.

Q 8. 30630558  Which of the following options captures the meaning of the last sentence best?

a)  Young people are only able to understand broad explanations.

b)  Young people fail to understand subtle shifts in meaning.


c)  Young people are bombarded with overt explanations.

d)  Young people are only comfortable with a general view of life.

Q 9. 30630558  What does the word 'inducement' as used in the passage, mean?

a)  Subtlety b)  Cruelty c)  Persuasion d)  Forceful

Q 10. 30630558  Which of the following does the author feel that society is unaware of?

a)  Instagram like quality that our lives have become. b)  The shift in direction forced upon our hearts and minds.

c)  The desire to shun light and reside in darkness. d)  The urge to exist in narrow and dingy conditions.

Directions for questions 1 to 30: Read the passages given below and answer the questions that follow.

Passage – 3

The one and only Khushwant Singh: "Aap Urdu seekhna chahte hain to ishq kar lijiye, Aur agar aap ishq karna chahte hain to Urdu seekh lijiye. (If
you want to learn Urdu, fall in love, if you want to fall in love, learn Urdu.)"All those who understand the power of language believe Urdu to be a
zubaan of grace and romance. Its elegance and beauty lie in its expression, rhythm and intellectual richness. It is a language of poets and
commoners alike.

Saadat Hasan Manto was of the view that a language cannot be made or developed, it develops on its own. According to Manto, no human
endeavour can destroy a language.

Perhaps Manto meant that a language is comparable to resistance. It finds its own path despite obstacles. It flows like water in a gushing stream. All
you require is the picturesque landscape and capacity to hold it. As long as people continue to read, speak and write a particular language, no power
on earth can demolish it.

Manto was of Kashmiri descent. He grew up speaking Punjabi in Samrala in Ludhiana. He was a school dropout and didn't score well in Urdu but
ended up as one of the finest short story writers in Urdu. His amazing stories on the horrors of mindless violence during the partition of Punjab and
Bengal in 1947 were wedded to reality. Is Urdu in decline in India? Can those crying the "death of Urdu" be described as alarmists? Or, is the
language actually thriving as some claim?

According to a recent report, the Delhi Police have asked officials to stop using certain Urdu and Persian words while registering first information
reports (FIRs). A notice to this effect was issued on April 11. Some of the nearly 400 Urdu words dropped include "intezaam", "dastavez", "guzarish",
"intiqam", "mulzim", "muj-rim" (arrangement, document, request, revenge, suspect, convict). All these words are commonly used, even in television
serials or in radio dramas and newspaper reports. Still, the argument being made is to "simplify" the FIR for the "common man"! Who is this
"common man" who struggles to understand the meanings of the words mujrim and mulzim?
Where have we arrived?

One way of answering these questions is through a sardonic verse by Akhtar Shahjahanpuri: "Abhi tehzeeb ka na noha na likhna, abhi kuch log
Urdu bolte hain (Do not write dirges of Urdu or culture, some people still speak Urdu)." Or, have a look at the following verse by an anonymous poet:
"Saliqe se hawaaon main jo khushboo ghol sakte hain abhi kuch log baqi hain jo Urdu bol sakte hain. (Those who can immerse fragrance in the air
with finesse, there are still those who can speak Urdu fluently.)

A humorist in Kashmir once jokingly made the point that as long as people continued hurling the choicest invectives in a particular language, spoke
in the same language in their dreams and nightmares, and fell in love while reading out poems in the same language, it was impossible for that
language to disappear.
Q 11. 30630558  What is the main idea of the passage regarding the status of Urdu language in India?
a)  The Delhi Police's decision to drop certain Urdu and Persian words in FIRs signifies the decline of Urdu in India.

b)  Saadat Hasan Manto believed that a language develops on its own and cannot be destroyed by human endeavors.
c)  Despite concerns over the decline of Urdu, the language continues to thrive and is cherished by those who speak it.

d)  The use of Urdu in literature and cultural expressions, such as poetry and dramas, is evidence of its enduring significance.

Q 12. 30630558  What can be inferred from the last paragraph regarding the status of a language?

a)  The continued use of a language in everyday life, dreams, and expressions of love ensures its preservation.

b)  The disappearance of a language is inevitable if it is not widely spoken or used in cultural expressions.
c)  Hurling invectives in a particular language contributes to the growth and popularity of that language.

d)  The preservation of a language depends solely on its literary contributions and cultural significance.

Q 13. 30630558  What is the tone of the passage regarding the status of Urdu language?

a)  Dismissive b)  Reflective c)  Sardonic d)  Optimistic

Q 14. 30630558  What does the word "resilience" mean as used in the passage?

a)  Fragility b)  Persistence c)  Flexibility d)  Sensitivity

Q 15. 30630558  Based on the information in the fifth paragraph, what can be inferred about Saadat Hasan Manto's journey as a writer?

a)  Saadat Hasan Manto faced numerous challenges in becoming a successful Urdu writer.
b)  Saadat Hasan Manto's early struggles with Urdu hindered his career as a writer.

c)  Despite his limited formal education, Saadat Hasan Manto emerged as a renowned Urdu writer.

d)  Saadat Hasan Manto's Punjabi background influenced his choice to write in Urdu.

Directions for questions 1 to 30: Read the passages given below and answer the questions that follow.
Passage – 4

It's been three months since the Sports Ministry put in place a high-profile probe panel - chaired by boxing legend MC Mary Kom and Olympic
medalist Yogeshwar Dutt as one of the members - to look into the sexual harassment allegations against Singh but a full report is still awaited. Not
casting aspersions on the credibility of the country's sporting greats but a disclaimer at this stage of the piece wouldn't be out of place. Like the ruling
party sitting MP Singh, both Usha and Mary Kom had been nominated to Rajya Sabha by the BJP. Dutt has fought elections for BJP and continues
to be the party's star campaigner.

The government did share what it called the "major findings" of the report prepared by the Mary Kom committee but was mysteriously silent on the
core issue - the serious charges against the sitting MP Singh. Going to Delhi Police too didn't help much. The detailed complaints, with specific time
and place, by wrestlers too didn't move the cops. They dragged their feet, dilly-dallied in filing FIR, till the Supreme Court stepped in.

Pushed to the corner, the wrestlers moved to Jantar Mantar - capital's 'last chance' island that gives space to those who have lost all hope. At this
point, the need of the hour for the IOA, the umbrella organisation for all sports, was to reach out to the wrestlers. The onus was also on Usha to win
back the confidence of her fellow athletes - not just the Olympians giving interviews to media but more importantly the minor girl living the trauma of
repeating her sexual harassment ordeal.

Usha, the 80s sprint queen, belongs to the era when athletes didn't have forums to air their grievances. Back in the day, the federations' office-
bearers were obliged to constitute only one committee that did the selection. POSH and ICC were abbreviations that hadn't been coined. Many
former athletes, well after their retirement, have spoken about the nightmare of being a young woman with a sporting dream. Being all alone at
academies and travelling to distant lands with male coaches, they would often face ugly situations. Some fearing backlash would swallow pride and
learn to live with abuse. Others would give up on their dreams, get married or pursue careers where they would be misfits.

Had Usha taken a short walk from her Rajya Sabha MP accommodation to Jantar Mantar, she would have understood the gravitas of the moment.
Lending ears to those on the fringes of the protest would have educated her about what this means to the wrestling fraternity.
Q 16. 30630558  What is the assumption behind the author's aspersion regarding some of the country's sporting greats?

a)  They are overrated by the Indian athletic fraternity.


b)  They could be biased due to their proximity with the government.

c)  They have internalised abuse and is insensitive towards those who face it.

d)  They are unaware of the changes that the sports have gone through with regards to tolerating abuse.

Q 17. 30630558  "The detailed complaints, with specific time and place, by wrestlers too didn't move the cops." - in elaboration of this sentence,
which of these options follows?

a)  Police are wary of taking steps against someone who is in a position of political power.

b)  Police are wary of bypassing the sporting greats and come to a decision on their own.
c)  Police are wary of going against the IOA who have already established a committee to investigate the complaints.

d)  Police are wary of taking steps against someone close to PT Usha.

Q 18. 30630558  Being all alone at academies and travelling to distant lands with male coaches, they would often face ugly situations. Which of the
following is the most convincing conclusion from this statement?

a)  Athletes are underexposed in practical conduct and often feel alienated while traveling in foreign lands.

b)  The athletes due to lack of protection from sporting bodies would not venture out in foreign lands.

c)  Athletes faced abuse from their senior coaches and had no outlet to address them.

d)  The academies were lonely places which felt alienating to the athletes.

Q 19. 30630558  Which of the following would the author agree to be the correct stance taken by Usha?

a)  Establishing a probe panel to investigate the accusation against MP Singh.

b)  Hearing out the grievance of young athletes in Jantar Mantar.

c)  Understanding the perils of being a young woman among male coaches.

d)  Forcing the cops to file an FIR against MP Singh.

Q 20. 30630558  Which of the following categories best describes this piece of writing?
a)  Fiction b)  Newspaper Article c)  Non-fiction essay d)  Poem

Directions for questions 1 to 30: Read the passages given below and answer the questions that follow.

Passage – 5

In a recent article (IE, April 12, "The path of excellence"), R Subrahmanyam, a former secretary, higher education, addresses the issue of what is
holding back India's universities. The views of civil servants, publicly expressed, are important, for bureaucrats exert a significant influence on how
India's public institutions of higher education are run. Subrahmanyam identifies "leadership" as crucial to achieving world class universities in India, a
goal expressed in the UGC (Institutions of Eminence Deemed to be Universities) Regulation Act 2017. There can be no quarrel with this, but two
points may be made on the suggestion. Do the central and state governments even aim to appoint persons of proven academic leadership to the
universities over which they exercise control?

Actually, they appear in their choice to be plainly guided by the expectation of political allegiance over all else. It would be difficult to identify too
many academics of proven excellence among the leadership appointments in India's public educational institutions in the recent past. Exceptions
apart, the myriad private institutions in the country do not have a better record. The second point is that even the best leaders can achieve nothing if
they are governed by rigid externally set rules. In India's higher education ecosystem, these rules appear mainly in the form of the ubiquitous UGC
guidelines.

For Indian universities to have even a fighting chance of being a player on the global stage requires governance approximating world standards. I
shall mention here a few areas in which India is off the global benchmark by a wide margin. Faculty everywhere are expected to teach and do
research.

In India, in neither of these areas is performance subject to a professional review. In the case of teaching, we know exactly what is needed. Courses
should be evaluated by students for content and delivery. While we may not want to rely on such evaluations entirely, they are a crucial means of
assessing teaching. The absence of student evaluation is the reason why in many of India's institutions students complain that teachers get away
with shoddy work or, worse still, with just not turning up in class. Research evaluation is a more difficult task and existing methods remain contested
even globally, but one thing is clear - the current practice in India's universities based on the UGC's Academic Performance Indicators (API) is flawed
beyond repair.

Scoring of publications according to where a paper has been published is known to be misleading when it comes to judging the impact of research
on the production of knowledge. The practice of numerical scoring of research output must be jettisoned for a more holistic approach. Moreover, it is
not clear whether some of the activities counted under the API system should be counted at all in an evaluation of academic output. This is apart
from the burden that scoring places on India's universities in terms of time and resources. Global best practices in the evaluation of academic
performance are known, and India should take on board the best aspects. In fact, it should, if Indian academics are to face a level playing field
internationally. It is the rules governing research rather than funding that is key to research output in Indian universities, though in some areas of
science and technology funding could make a difference.
Q 21. 30630558  What is the main concern raised by R Subrahmanyam regarding the state of Indian universities?

a)  The lack of academic leadership in university appointments

b)  The influence of rigid external rules on university governance

c)  The absence of professional review in evaluating teaching and research

d)  The need for funding to improve research output in universities

Q 22. 30630558  Which of the following is the most suitable title for the passage?

a)  The Significance of Language in Indian Culture

b)  The Role of Bureaucracy in Indian Higher Education

c)  The Importance of Academic Evaluation in Indian Universities

d)  The Need for Global Standards in Indian University Governance

Q 23. 30630558  Which of the following words from the passage is synonymous with "hinder"?

a)  Exert b)  Proven c)  Approximating d)  Obstruct

Q 24. 30630558  Based on the last paragraph, which of the following can be inferred about research output in Indian universities?
a)  Research output in Indian universities is solely dependent on funding.

b)  The evaluation of research output in Indian universities is flawed and needs improvement.

c)  Indian universities prioritize research rules and regulations over funding.

d)  Indian universities have achieved a level playing field internationally in terms of research output.

Q 25. 30630558  Which of the following statements, if true, would strengthen the author's claim about the need for improved leadership in Indian
universities?

a)  Several universities in India have recently appointed renowned academic leaders to key leadership positions.

b)  
The UGC (Institutions of Eminence Deemed to be Universities) Regulation Act 2017 mandates the appointment of leaders with proven academic
excellence.
c)  
A study conducted by a reputable research institute found a positive correlation between effective leadership and research output in Indian
universities.
d)  The state and central governments have allocated significant funding towards leadership development programs for university administrators.

Directions for questions 1 to 30: Read the passages given below and answer the questions that follow.

Passage – 6

A visit to Kashi, among the oldest cities of the world, is deeply personal. For Kashi, or Varanasi, is at the confluence of life and death and all that
happens in between. Kashi is the city of faith. We come in search. We come to make amends. We come to seek forgiveness. We come in
remembrance. We come in acceptance. We come in peace. We come to pay our respects, to express our gratitude. We come in prayer.

As the cradle of Hinduism, Kashi is meant to be unifying; the cornerstone of an ancient syncretic culture that, despite its many contradictions, seeks
to bind. Kashi's flavour of love and devotion is nurtured by the once mighty Ganga still flowing alongside majestically. She's been there throughout,
bearing witness, and cleansing our sins. Having done so for centuries, she is now tired and weary. She has shrunk, filled to the gills with all the muck
we have shed.

And lording over Kashi and its many avatars is The Lord himself. As Vishwanath, He was till recently quietly ensconced at his abode as if in benign
emphasis of his unfussy nature. A little water was enough. Maybe a simple garland of akundo phool tied together loosely. That's all he ever wanted.

Now He is bathed in LED brilliance and marble caked opulence. The spectacular arati on the banks of the Ganga is a sound-and-light show with
music blaring from loudspeakers. Thousands throng the ghats every evening for a piece of this never-to-be-missed event. Their cellphones in focus,
lemon tea is a vendor away. Business is brisk.

On the way to Dashashwamedh Ghat, hundreds walk along, only to stop at times to pick up one of those colourful shirts with Har Har Mahadev
calligraphy. Some buy bangles for the wife and daughter. Others a dug-dugi to silence that bawling boy who has probably been scarred for life at the
sheer size of the crowd in procession. Many even find the time to grab a quick plate of Varanasi's famous chaat.

Nearby, a short flight of steps down, is Harmony Book Store where James Prinsep shares space with, among many others, Geetanjali Shree, Annie
Ernoux and Ismat Chughtai. Fiercely independent, these women of elevated thought have paved a path for themselves and the world. Prinsep's
album of sketches titled 'Benares Illustrated' is a paean to the Kashi of yore, black and white depictions of who we were and where we came from.
As for what we have become, the crow conference in Shree's 'Tomb of Sand' offers even more pointers.
Q 26. 30630558  Why does the author mention the statue being bathed in LED light?

a)  To show the effects of climate change. b)  To show the material opulence that has gripped Kashi.

c)  To show the love and devotion that the Lord demands. d)  To show the development that India has undergone recently.

Q 27. 30630558  Which of the following does not apply to Kashi?

a)  A town which merges things. b)  A town upholding the cycle of life.

c)  A place laden in faith. d)  The town is unfussy in nature.

Q 28. 30630558  What does 'ensconce' mean in the passage?


a)  Settle b)  Prohibit c)  Enumerate d)  Demarcate

Q 29. 30630558  Which of the following would the author agree to be valid reason(s) for visiting Kashi?

a)  To revise life. b)  In hope of affirmation. c)  Both (a) and (b) d)  Neither (a) nor (b)

Q 30. 30630558  All the words below are related in meaning, except:

a)  Opulence b)  Remembrance c)  Brilliance d)  Majestically

Current Affairs Including General Knowledge


Passage – 1

Calling upon Shanghai Cooperation Organisation (SCO) member states to root out terrorism collectively and fix accountability on its supporters,
Defence Minister [1] asserted that any kind of terrorist act or support to it is a major crime against humanity. "Peace and prosperity cannot coexist
with this menace," [1] added while chairing the Defence Ministers' meeting which India is hosting for the first time since it became a full SCO
member in [2].

Separately, in a statement on bilateral talks between Chinese Defence Minister Li Shangfu and [1], China said that the two countries share far "more
common interests than differences" and the two sides should take a "long-term view, place the border issue in an appropriate position in bilateral
relations, and promote the transition of the border situation to normalized management".

"If a nation shelters terrorists, it not only poses a threat to others, but for itself too. Radicalisation of youth is a cause of concern not only from the
point of view of security, but it is also a major obstacle in the path of socioeconomic progress of society. If we want to make the SCO a stronger and
more credible international organisation, our topmost priority should be to effectively deal with terrorism," [1] said.
Q 31. 30630558  Who is the current Defence Minister of India whose name has been redacted with [1] in the passage above?

a)  Rajnath Singh b)  Amit Shah c)  Dr. Subrahmanyam Jaishankar d)  Piyush Goyal


Q 32. 30630558  India became a full Shanghai Cooperation Organisation (SCO) member in [2], which of the following years has been redacted with
[2] in the passage above?

a)  2011 b)  2015 c)  2017 d)  2018

Q 33. 30630558  India and Pakistan were inducted as full members in the Shanghai Cooperation Organisation in Astana summit. Astana is the
capital city of___________.

a)  Kyrgyzstan b)  Kazakhstan c)  Tajikistan d)  Turkmenistan

Q 34. 30630558  Which of the following countries has become the Shanghai Cooperation Organization dialogue partner in March 2023?

a)  UAE b)  Bangladesh c)  Saudi Arabia d)  Iraq

Q 35. 30630558  Which of the following countries is Not an observer country in the Shanghai Cooperation Organization (SCO)?

a)  Afghanistan b)  Belarus c)  Mongolia d)  Armenia

Q 36. 30630558  Which of the following is the only Central Asian country Not a part of full member in Shanghai Cooperation Organization (SCO)?

a)  Turkmenistan b)  Kyrgyzstan c)  Kazakhstan d)  Tajikistan

Q 37. 30630558  In 2021, the decision was made to start the accession process of _________to the SCO as a full member.

a)  Italy b)  Iran c)  Iraq d)  Nepal

Passage – 2

BSP's Afzal Ansari was disqualified as a member of the Lok Sabha after he was convicted and sentenced to fouryear imprisonment in a kidnapping-
and-murder case by a court in Uttar Pradesh. Ansari, the Lok Sabha member from [1], was sentenced to four years in jail by an MP/MLA court on
Saturday. His brother, criminal-turned-politician Mukhtar Ansari, was also convicted in the same case and was sentenced to 10 years in jail.

The brothers were booked under the UP Gangsters Act in connection with the murder of the then MLA from [1], Krishnanad Rai, on November 29,
2005 along with the kidnapping and murder of Varanasi-based trader Nand Kishore Rungta in 1997.

"Consequent upon his conviction…Afzal Ansari, Member of Lok Sabha representing the [1] Parliamentary Constituency of Uttar Pradesh, stands
disqualified from the membership of Lok Sabha from the date of his conviction i.e. 29 April, 2023 in terms of the provisions of Article 102(1)(e) of the
Constitution of India read with Section 8 of the Representation of the People Act, 1951," according to a Lok Sabha secretariat notification.

The incident sparked tension in the area with the residents of Keshavnagar observing a bandh to condemn the murder and demanding stern action
against the killers. Congress leader [2] recently lost his Lok Sabha membership after he was convicted in a 2019 defamation case and sentenced to
two years in jail.
Q 38. 30630558  BSP's Afzal Ansari was disqualified as a member of the Lok Sabha after he was convicted and sentenced to four-year
imprisonment in a kidnapping-and-murder case by a court in Uttar Pradesh. Afzal Ansari was the Lok Sabha member from [1]. Which of the following
constituencies has been redacted with [1] in the passage above?

a)  Agra b)  Ghazipur c)  Lucknow d)  Kanpur

Q 39. 30630558  In March 2023, Name the Congress leader who lost his Lok Sabha membership after he was convicted in a 2019 defamation case.

a)  Kapil Sibal b)  Jairam Ramesh c)  Shashi Tharoor d)  Rahul Gandhi

Q 40. 30630558  According to the Section 8 of the Representation of the People Act, 1951, if an MP or MLA is convicted for any other crime and is
sent to jail for __________ years or more, he/ she will be disqualified for 6 years from the time of release.

a)  2 years or more b)  3 years or more c)  4 years or more d)  5 years or more

Q 41. 30630558  Under the Representation of the People Act, 1951, the question of disqualification of a member of state legislature is decided
by________.

a)  State Election Commission b)  Governor of the State

c)  State Legislative Assembly Speaker d)  President of India


Q 42. 30630558  On 10 July 2013, the Supreme Court of India, in its judgment of the__________, decided that any MP, MLA or MLC who is
sentenced for a crime and granted at least two years of imprisonment, loses membership of the House with immediate effect.

a)  Prakash Singh v. Union of India case b)  K.M Nanavati v. Union of India case

c)  Om Prakash v. Union of India case d)  Lily Thomas v. Union of India case

Q 43. 30630558  Under the Representation of the People Act, 1951, the question of disqualification of a member of members of Parliament is
decided by________.

a)  President of India b)  Lok Sabha Speaker c)  Prime Minister d)  Vice President of India

Q 44. 30630558  Which of the following is/are the grounds for disqualification for the membership under the Representation of the People Act, 1951?

a)  If he/she has been convicted for promoting enmity between different groups or for the offence of bribery.

b)  If he/she has been punished for preaching and practising social crimes such as untouchability, dowry and sati.

c)  If he/she has been dismissed from government service for corruption or disloyalty to the State.

d)  All of the above

Passage – 3

The Reserve Bank of India (RBI) announced the introduction of the SDF as the basic tool to absorb excess liquidity under the new monetary policy.
The SDF will help the central bank in absorbing liquidity (deposits) from commercial banks without giving government securities in return to the
banks.

Governor Shaktikanta Das said that the SDF will be at 3.75 percent, 0.25 percentage points below the repo rate, and 0.50 percentage points lower
than the marginal standing facility (MSF) which helps banks with funds when required.

When the central bank has to absorb a tremendous amount of money from the banking system through the reverse repo window, it becomes difficult
for it to provide the required volume of government securities in return. This happened during the time of demonetisation. In this sense, the SDF is a
collateral-free arrangement meaning that RBI need not give collateral for liquidity absorption.

The idea of an SDF was first mooted in the [1] Monetary Policy Committee report in 2014, which later received the government's nod following an
amendment to the RBI Act in 2018, vide the Finance Bill. Since then, the central bank has proposed introducing the SDF for liquidity management so
that banks can park as much money with it as they want without getting collateral, and at a lower rate than the reverse repo rate. The RBI again
dusted off the idea of introducing the SDF when the pandemic hit in 2020 too but no decision was taken.
Q 45. 30630558  What is the full form of SDF in the passage above?

a)  Stable Deposit Facility b)  Standing Deposit Facility c)  Static Deposit Facility d)  Statutory Deposit Facility

Q 46. 30630558  The idea of an SDF was first mooted in the [1] Monetary Policy Committee report in 2014, which later received the government's
nod. What is the name of the committee chairman redacted with [1] in the passage above?

a)  Raghuram Govind Rajan b)  Duvvuri Subbarao c)  Viral V. Acharya d)  Urjit Patel

Q 47. 30630558  Which of the following acts has been amended by the Finance Act, 2016, to provide for a statutory and institutionalised framework
for a Monetary Policy Committee, for maintaining price stability, while keeping in mind the objective of growth?

a)  Reserve Bank of India Act, 1934 b)  Reserve Bank of India Act, 1947
c)  Reserve Bank of India Act, 1949 d)  Reserve Bank of India Act, 1969

Q 48. 30630558  What will happen if RBI increases Repo Rate?

a)  It can lead to lower borrowing costs for businesses and individuals.

b)  It can lead to higher borrowing costs for businesses and individuals.

c)  It can lead to more liquidity in the market.

d)  There is no effect on borrowing costs for businesses and individuals

Q 49. 30630558  What will happen if RBI increases Bank rate?

a)  It will make the loans more expensive for the commercial banks.
b)  It will make the loans less expensive for the commercial banks.

c)  It will make the more liquidity in the market.


d)  There is no effect on commercial banks.

Q 50. 30630558  Which of the following is/are the monetary measure that can be taken to address inflation in the economy?

a)  Decrease Reverse Repo rate b)  Increase Statutory Liquidity Ratio


c)  Decrease the Bank rate d)  All of the above

Q 51. 30630558  Which of the following statements is/are true regarding the SDF?

a)  In 2018, the amended Section 17 of the RBI Act, 1934 empowered the RBI to introduce the SDF.

b)  The main purpose of SDF is to reduce the excess liquidity in the system, and control inflation.

c)  Both (a) and (b) Statements are true


d)  Neither (a) nor (b) Statements are true

Passage – 4

The much-anticipated Climate Change 2023: Synthesis Report is based on years of work by hundreds of scientists during the IPCC sixth
assessment cycle which began in 2015. The report provides the main scientific input to COP28 and the Global Stocktake at the end of this year,
when countries will review progress towards the Paris Agreement goals.

The report reiterates that humans are responsible for all global heating over the past 200 years leading to a current temperature rise of 1.1°C above
pre-industrial levels, which has led to more frequent and hazardous weather events that have caused increasing destruction to people and the
planet. The report reminds us that every increment of warming will come with more extreme weather events.

The report outlines that the 1.5°C limit is still achievable and outlines the critical action required across sectors and by everyone at all levels. The
report focuses on the critical need for action that considers climate justice and focuses on climate resilient development. It outlines that by sharing
best practices, technology, effective policy measures, and mobilising sufficient finance, any community can decrease or prevent the usage of carbon-
intensive consumption methods. The biggest gains in well-being can be achieved by prioritizing climate risk reduction for low-income and
marginalized communities.
Q 52. 30630558  What is the full form of IPCC mentioned in the passage above?

a)  International Panel on Climate Change b)  Intergovernmental Panel on Climate Change

c)  Intergovernmental Proposal on Climate Change d)  Intergovernmental Panel on Climate Convention

Q 53. 30630558  The IPCC is the UN body for assessing the science related to climate change. It was set up in 1988 by the World Meteorological
Organization and ___________.

a)  United Nations Environment Programme b)  United Nations Framework Convention on Climate Change
c)  Food and Agriculture Organization d)  World Bank

Q 54. 30630558  Which of the following statements is/are true regarding the IPCC?

a)  It is an intergovernmental body of the United Nations.

b)  Its job is to advance scientific knowledge about climate change caused by human activities.

c)  The IPCC does not conduct its own original research.

d)  All of the above

Q 55. 30630558  The United Nations Conference on Environment and Development (UNCED), also known as the 'Earth Summit', was held from 3-
14 June 1992 in________.

a)  Kyoto b)  Tokyo c)  Rio de Janeiro d)  Geneva

Q 56. 30630558  Earth Day is an annual event on__________.

a)  March 21 b)  March 22 c)  April 21 d)  April 22


Q 57. 30630558  The 6th Assessment Report (AR6) assesses scientific, technical, and socio-economic information concerning climate change.
Which of the following is/are the messages for policymakers recommended by IPCC?

a)  Current policy action will lead to further temperature rise, and the impacts on humans and other forms of life will become more severe.

b)  At current emissions levels, we will deplete the remaining carbon budget (of 500 GtCO2).
c)  We need to cut GHG emissions across all sectors urgently, within this decade and no later.

d)  All of the above

Q 58. 30630558  Stockholm+50 is held in Stockholm in 2022. It commemorated the 50 years since the 1972 United Nations (UN) Conference on the
Human Environment (also known as the Stockholm Conference). Stockholm is a city in_________.

a)  Norway b)  Denmark c)  Switzerland d)  Sweden

Passage – 5

President Joe Biden expressed his "solidarity" with [1] Muslims in China and [2] Muslims in Myanmar in a Ramadan message to American Muslims.
"Together with our partners, the United States stands in solidarity with Muslims who continue to face oppression- including [1] in the People's
Republic of China, [2] in Burma, and other Muslim communities facing persecution around the world," the statement said.

The statement was released on 22 March, which marked the first night of Ramadan for many in the US and across the globe. "During this sacred
time of reflection, the United States also reaffirms our support to Muslim communities suffering hardships and devastation," the statement said. "We
will continue to stand with the people of Turkiye and Syria-who have lost many loved ones during the recent devastating earthquakes-and with the
people of Pakistan, who are rebuilding their lives following last summer's floods."

"Today especially, we remember the universal human right to practice, pray, and preach our faiths peacefully and openly." China's persecution of the
[1] has been documented by human rights organisations and by the UN's Office of the High Commissioner for Human Rights and has been widely
condemned by western governments.
Q 59. 30630558  Who among the following Muslim ethnic communities from China has been redacted with [1] in the passage above?
a)  Moors b)  Rohingyas c)  Pastun d)  Uighurs

Q 60. 30630558  Who among the following Muslim ethnic communities from Myanmar has been redacted with [2] in the passage above?

a)  Rohingyas b)  Hamas c)  Berbars d)  Uighurs

Q 61. 30630558  The world's largest refugee camp, home to over 700,000 [2] refugees from Myanmar is located in:

a)  India b)  China c)  Bangladesh d)  USA

Q 62. 30630558  The United Nations High Commissioner for Refugees (UNHCR) is a UN agency mandated to aid and protect refugees, forcibly
displaced communities, and stateless people, and to assist in their voluntary repatriation, local integration or resettlement to a third country. It is
headquartered in:

a)  New York b)  Paris c)  London d)  Geneva

Q 63. 30630558  In January 2020, The International Court of Justice (ICJ) has given its verdict on the [2] crisis. Which of the following statements is
Not correct?

a)  The ruling of the ICJ is not binding on Myanmar, and can be appealed.

b)  However, no means are available to the ICJ to enforce it.

c)  
The government of Myanmar should immediately take "all measures within its power" to prevent atrocities against members of the minority [2]
Muslim community.
d)  This is to be done in accordance with its obligations under the Convention on the Prevention and Punishment of the Crime of Genocide.

Q 64. 30630558  The first Indian to become a permanent judge of International Court of Justice (ICJ) was_________.

a)  Sir Benegal Narsingh Rau b)  Justice Nagendra Singh c)  R. S. Pathak d)  Dalveer Bhandari

Q 65. 30630558  Who among the following Indians served as the President of the International Court of Justice (ICJ)?
a)  Sir Benegal Narsingh Rau b)  R. S. Pathak c)  Justice Nagendra Singh d)  Dalveer Bhandari

Legal Aptitude
Directions for questions 66 to 105: Each set of questions in this section is based on the reasoning and arguments, or facts and principles set out
in the preceding passage. Some of these principles may not be true in the real or legal sense, yet you must conclusively assume that they are true
for the purposes of this Section. Please answer each question on the basis of what is stated or implied in the corresponding passage. Do not rely on
any principle of law other than the ones supplied to you, and do not assume any facts other than those supplied to you when answering the
questions. In some instances, more than one option may be the answer to the question; in such a case, please choose the option that most
accurately and comprehensively answers the question.
Disclaimer
All names and facts mentioned are hypothetical and are being used for educational purposes. Any resemblance with any person, institution or
situation is purely coincidental and unintentional.

Passage – 1

Article 21 of the Indian Constitution reads, 'Protection of life and personal liberty - No person shall be deprived of his life or personal liberty except
according to procedure established by law'. It is available to both citizens as well as non-citizens. It is one of the most judicially enlarged articles of
the Indian Constitution. Different judicial interpretations have given varying meaning as to the constituents of the right to life. For instance, in the
case of Maneka Gandhi v. Union of India, in the year 1978, it was held that right to life is not confined to physical existence but it includes within its
ambit the right to live with human dignity. Accordingly, the procedure which abridges rights under Article 21 must be just, fair, and reasonable. It
cannot be arbitrary depending upon the whims and fancies of the executive. In the case of KS Puttaswamy v. Union of India, over-ruling ADM
Jabalpur, the right to privacy has been held as forming an essential tenet of right to life. The judgment reasoned that privacy forms the core human
values of dignity and freedom. Liberty is of true substance only when enjoyed with dignity. State should strike a balance between privacy of citizens
and the legitimate concerns of the state.
Q 66. 30630558  The government of Assam enacted a law which prohibited any employee from wearing jeans and other casual attire like T-shirts,
slippers in government offices. The reason the government gave was that government offices must maintain a decorum and dignity and casual wear
must not be worn by employees during office hours. Decide.

a)  The ban of the Assam government is reasonable, just, and fair.

b)  The ban by the Assam government is unreasonable and unfair.

c)  The ban on attire is a direct violation of Article 19 of the Indian Constitution.

d)  Government should not interfere in the dressing etiquettes of its employees.

Q 67. 30630558  Nisha gets to know that her Instagram messages are being monitored by 'X' agency of the Central government. Mr. K, who is an
officer with 'X' agency is spying on Nisha, whom he loved but Nisha rejected his proposal. Decide.

a)  Nisha should delete her Instagram account permanently.

b)  Nisha's Article 21 is violated.

c)  Nisha should file an FIR against Mr. K.


d)  Nisha should accept Mr. K for marriage. In today's deceitful world where would she find such a loving, caring, and possessive guy.

Q 68. 30630558  Rahul is accused of molestation by a lady, Gamini. The police acting upon a complaint, go to his house and arrest him. While
arresting Rahul, a police officer Shyam slaps Rahul. Another police officer holds his hair and slaps him again. He is asked to take off his pants and
walk in his locality in her undergarments. Decide.

a)  Rahul's right to life is violated by the police as his arrest is not made in a dignified manner.
b)  Rahul's right to life is violated as he should have been informed the grounds of his arrest.

c)  Rahul's right to life is not violated as he is a molester.

d)  Rahul's right to life is not violated as the cruel treatment by the police will set a deterrent in the society, so that in future, no such events occur.

Q 69. 30630558  The Indian secret agencies have credible and vetted inputs that Vinod, an Indian national, has got plans to put bombs in major
cities of India. They spy into his wats app messages and twitter account and read all messages he sent and received. Decide.

a)  Vinod must be given capital punishment to set a deterrent.


b)  Vinod must be sent to Pakistan as he is an anti-national.

c)  The actions of the Indian secret agency are justified in law.


d)  The actions of the Indian secret agency are not justified in law.

Q 70. 30630558  A law was enacted by a state government of India which prohibited the residents of that state from eating non-vegetarian food. No
reason was assigned for taking such a step. Decide.

a)  The law is not violative of Article 21 as non-vegetarian food is not good for health.

b)  The law is violative of Article 21 as it is arbitrary.


c)  The law is violative of Article 25 of the Constitution which allows freedom of religion.

d)  The law is not violative of any article of the Constitution.

Directions for questions 66 to 105: Each set of questions in this section is based on the reasoning and arguments, or facts and principles set out
in the preceding passage. Some of these principles may not be true in the real or legal sense, yet you must conclusively assume that they are true
for the purposes of this Section. Please answer each question on the basis of what is stated or implied in the corresponding passage. Do not rely on
any principle of law other than the ones supplied to you, and do not assume any facts other than those supplied to you when answering the
questions. In some instances, more than one option may be the answer to the question; in such a case, please choose the option that most
accurately and comprehensively answers the question.
Disclaimer
All names and facts mentioned are hypothetical and are being used for educational purposes. Any resemblance with any person, institution or
situation is purely coincidental and unintentional.

Passage – 2

Article 11 of Vienna Convention on Law of Treaties is about ratification of a treaty. Ratification is an international act whereby a state consents to be
bound by a treaty on the international platform. An international treaty comes into force when ratified by the prescribed number of states. It is a good
practice to ratify a treaty in certain situations - when there is such an express provision in the treaty; or when the intention is evident from the talks
and circumstances; or when treaty is signed under the condition that ratification is necessary. A treaty may come into force even before ratification, if
the parties so intend. There is no general duty on the state to ratify a treaty as each state is sovereign. Ratification may also require some changes
in the state's domestic law. There are some benefits of ratification. It gives an opportunity to consider treaty in detail as international law related to
treaty is only a broad framework. Additionally, the treaty can be repudiated later, if the state desires so. Another benefit is that it gives the state an
opportunity to consider public opinion. Some disadvantages of not ratifying an internationally signed treaty are that is strikes at the reputation or
positive image of such country, in international community. Also, the trustworthiness of such country diminishes. This leads to a loss of goodwill
which plays the role of soft power in international politics.
Q 71. 30630558  An international treaty was signed regarding measures to curb the scourge of terrorism. All countries of the world signed it.
Pakistan also signed it but did not ratify it. Decide.

a)  Pakistan may or may not ratify an international treaty as there is no such duty under international law.

b)  Pakistan has to ratify the treaty as there is a legal obligation over it.

c)  Pakistan has to ratify the treaty as it signed it in the first place.

d)  Pakistan cannot ratify the treaty as it is the hub of international terrorists.

Q 72. 30630558  Thailand and Afghanistan signed a bilateral treaty under which Thailand will train Afghani people on methods to improve tourism.
Both counties ratify the treaty. The government of Afghanistan is outed by Taliban insurgency. Decide.

a)  Thailand cannot repudiate the treaty under any circumstance whatsoever.

b)  Thailand can repudiate the treaty in its Parliament.


c)  Thailand has to be paid compensation.

d)  Afghanistan and Thailand will have to approach the United Nations for settlement of this issue.

Q 73. 30630558  Mexico, Brazil, Turkey, and China sign a treaty on 4th November, 2021. It was mutually decided that the treaty will come into force
on 4th November, 2022 provided each of the signatory country ratifies it. Decide.

a)  Suppose Mexico ratifies the treaty but not Brazil, Turkey, or China; then the treaty comes into force on 4th November, 2022.
b)  Suppose none of the four countries ratify the treaty; the treaty comes into force on 4th November. 2021.
c)  Suppose Brazil, Turkey and China ratify the treaty but not Mexico; then the treaty can never come into force.

d)  Suppose Mexico, Brazil, Turkey, and China ratify the treaty; then the treaty comes into force on 4th November. 2022.

Q 74. 30630558  India signs a climate change treaty. It seeks to ratify the same in its Parliament. According to the passage, what benefits would
India have?

a)  India will get climate finance from World Bank.

b)  India should ask the developed countries to first lead the talk and ratify the climate change treaty in their respective countries.

c)  India can seek the views of its intellectuals, academicians, bureaucrats, scientists, etc.
d)  India can get certain concession from developed countries.

Q 75. 30630558  USA and Russia sign a treaty on non-proliferation of nuclear weapons. USA honours it by ratifying it in its Senate. Russia does not
ratify it. Decide.

a)  Both Russia and USA will have to face the consequences under international law.

b)  Only a powerful country like Russia can dare to challenge USA authority.

c)  A negative image of Russia in the international forum will be created affecting its trustworthiness.
d)  A positive image of Russia will develop among the countries which think that USA is the world's most powerful country.

Directions for questions 66 to 105: Each set of questions in this section is based on the reasoning and arguments, or facts and principles set out
in the preceding passage. Some of these principles may not be true in the real or legal sense, yet you must conclusively assume that they are true
for the purposes of this Section. Please answer each question on the basis of what is stated or implied in the corresponding passage. Do not rely on
any principle of law other than the ones supplied to you, and do not assume any facts other than those supplied to you when answering the
questions. In some instances, more than one option may be the answer to the question; in such a case, please choose the option that most
accurately and comprehensively answers the question.
Disclaimer
All names and facts mentioned are hypothetical and are being used for educational purposes. Any resemblance with any person, institution or
situation is purely coincidental and unintentional.
Passage – 3

Provocation is a well recognised defence under criminal law. If a man on account of being provoked by another man, does a wrongful act which is
prohibited under Indian Penal Code (IPC), he can avail to himself the defence of provocation. In the landmark case of KM Nanavati v. State of
Maharashtra, it was held that not every act of instigation, or provoking can come within the defence of 'provocation'. To avail this defence, it has to be
proven that the provoking act was 'grave and sudden'. By 'grave', it is meant that the provoking act was serious enough to instigate a reasonable
man placed under similar circumstances to act. The man must lose the balance of his mind due to the provoking act such that there is no element of
premeditation. If a man is provoked but he is well within his senses, such that he can understand the consequences of that which he is about to act,
he cannot avail the defence of provocation. 'Sudden' means that there is no cooling off period between the provoking act and the subsequent act of
the accused. The time internal must not be too long. Provocation can be words, signs, symbols, or even by gestures.
Q 76. 30630558  Ramesh was returning home after sowing paddy in his fields. He sees Manoj, his arch-enemy, whistling and making lewd
comments on his beloved sister, Aarti. Ramesh hits Manoj with his ploughshare. Manoj dies. Decide.

a)  Ramesh can take the defence of provocation.


b)  Ramesh cannot take the defence of provocation as it was not grave.

c)  Ramesh cannot take the defence of provocation as beautiful girls do face lewd comments.

d)  Ramesh should file an FIR in the nearest police station.

Q 77. 30630558  Harris fell in a pot-hole on a road in Bengaluru. He was enraged and went to the house of the Road Minister of Karnataka and
threw a big stone on his house breaking the window panels. He claims provocation as a defence. Decide.

a)  Harris will fail as falling in the pot-hole cannot be considered a seriously provoking act.

b)  Harris will fail as his act is not reasonable.

c)  Harris will succeed as it is the duty of the Road Minister to maintain roads in good condition.

d)  Harris will succeed as he got injured due to falling in the pot-hole.


Q 78. 30630558  Karan, an Indian Air Force officer, was stationed in Sudan on a UN peace keeping operation. He is captured by the rebel groups
and kept alive. They do not disclose this fact. The Indian Air Force could not trace Karan and after a reasonable time, declared him dead. Meena,
Karan's wife is devastated. She has a son Rajbir, aged 8 years, with Karan. Tapan, Karan's younger brother marries Meena to fill in the emotional
void due to Karan's death. After a year, Meena gives birth to a daughter, Sonali. When Sonali turned 20 years of age, Karan escapes and comes
back home. On discovering what had transpired, he shoots Tapan, Meena, Rajbir, and Sonali. All four die. Decide.

a)  Karan cannot take the defence of provocation as it is his brother himself who married Meena and not any stranger.

b)  Karan cannot take the defence of provocation as the provocation was not grave.
c)  Karan can take the defence of provocation as the provocation was grave as well as sudden.

d)  Karan can take the defence of provocation as his own wife and brother had given birth to Sonali.

Q 79. 30630558  Bhavesh loved Disha. Both plan to get married after turning adults. Bhavesh got seriously interested in CLAT preparation and was
not able to keep in regular touch with Disha. Due to which there relationship lost the spark. Lovish got in touch with Disha and the two enter into a
relationship. Bhavesh on discovering about Lovish, thrusts a sword in Lovish's abdomen resulting in grievous injury. Decide.

a)  Disha is liable for provoking Bhavesh. b)  Bhavesh is liable for attempt to murder.

c)  Bhavesh is not liable for causing grievous injury to Lovish. d)  Bhavesh is liable for causing grievous injury to Lovish.

Q 80. 30630558  Rustom, a Navy officer, got stationed in Seychelles Island for 2 years. In the meantime, his wife Pavani, got emotionally attached to
Navtej, a friend of Rustom. Pavani has a daughter, Rekha, with Rustom. Rustom's impromptu leave got sanctioned and in order to surprise Pavani,
he plans to come to home straight away without informing Pavani about the leave. On coming home, he finds Pavani in physically intimate position
with Navtej, in their bedroom. Enraged, he instantly takes out his gun, and shoots Navtej and Pavani dead. Decide.

a)  Rustom can avail provocation defence.

b)  Rustom cannot avail provocation defence as the provocation was not grave.
c)  Rustom cannot avail provocation defence as he is a Navy officer and must know when to use a gun and when not.

d)  Rustom cannot avail provocation defence as the provocation was not sudden.

Directions for questions 66 to 105: Each set of questions in this section is based on the reasoning and arguments, or facts and principles set out
in the preceding passage. Some of these principles may not be true in the real or legal sense, yet you must conclusively assume that they are true
for the purposes of this Section. Please answer each question on the basis of what is stated or implied in the corresponding passage. Do not rely on
any principle of law other than the ones supplied to you, and do not assume any facts other than those supplied to you when answering the
questions. In some instances, more than one option may be the answer to the question; in such a case, please choose the option that most
accurately and comprehensively answers the question.
Disclaimer
All names and facts mentioned are hypothetical and are being used for educational purposes. Any resemblance with any person, institution or
situation is purely coincidental and unintentional.

Passage – 4

The copyright law aims to endorse exclusive forms of artistic works at the same time safeguarding the rights of its inventor. The scope of the law
extends as far as literary, music, software, graphics, choreography, movies and likewise is concerned. These categories are further sub-categorised
into books, documentaries, painting, articles, and likewise, which aim in promoting work and not ideas. It is to be noted that the work of the creator is
protected by the copyright law irrespective of its content or quality. Although the registration of the work is not a mandate, nevertheless it is always
advisable to register the work the moment it is put to force. This will help in case of future disputes, if any.

Now, infringement of a copyright work takes place when any other person other than the creator clutches or replicates the same work. It implies that
intellect and perceptions are not safeguarded but the idea's voice is protected. At times, infringement also takes place when the work is not directly
imitated but it is parallel to an illegitimately unoriginal effort. Nonetheless, there are exceptional clauses which expressly states that a work copied
from a creator will not be considered illegal if the creator has given consent explicitly. Fair use of imitation is still prevalent and is legal. Second
clause is whenever the copyright work has expired, anyone can reproduce the same work because after expiry, there is no license fee given to the
creator. Third clause is applicable on all works which cannot be copyrighted, for instance, ideas, titles, names, etc. Nonetheless, if a person copies
factual ideas in the form of a book, the exception will not prevail.

Earlier the concept of infringement of this law was not seen much, however, with its evolution, came new challenges. The owner of a thing so
copyrighted, deserves every single penny of appreciation owing to the hard work and diligence that has gone under creating a unique content.
Whenever a person infringes the copyright, the creator is bestowed with an opportunity to seek relief for the same, either in form of monetary
compensation for the loss incurred due to infringement or by paying for the license of the owner. Licensing is a lawful authorization given to another
person other than the creator to use all or any the copyrighted products of the owner. The owner always can go for discovering multiple licensing
options to secure his financial gain earned from the other party's usage of the products. Licensing is considered as evidence too, in court.

Source: https://www.mondaq.com/india/copyright/1185696/copyright-law-in-india
Q 81. 30630558  Y made a movie depicting a famous slum area of India. Z, who was also a movie director, came up with a movie that was based on
the struggles of a middle-class family. Coincidentally, both these movies had one character's name in common: Ganga Raam. Now, Y wishes to sue
Z for copying an element from his book, considering that such names are rarely used in a movie these days. Which of the following options would be
a suitable decision in this case?

a)  Z is liable for infringement since the specific name is already used in Y's movie.

b)  Z is liable for infringement since the specific name is rarely used in movies these days.

c)  Z is not liable for infringement since names cannot be copyrighted.

d)  Z is not liable for infringement since he has not copied the entire movie.

Q 82. 30630558  M was a famous artist, who had written a book based on a very creative fictional character. Now, he agreed (informally) that he'll
pay some royalty to N, who will be helping him to market his book and enhance sales. A couple of months later, M released a video that got
immense popularity, as it was based on a similar character as the book. Now, N wishes to sue him because the video was made without his
knowledge. Further, he states that the work has been reproduced illegally without the permission of relevant stakeholders.

a)  M is guilty of reproducing the work illegally since he did not honor his agreement with N.

b)  M is not guilty of reproducing the work illegally since there was no such liability on him to consult N.

c)  M is guilty of reproducing the work illegally since he should have shared the revenue from that video with N.
d)  M is not guilty of reproducing the work illegally since N is merely a minor stakeholder in this transaction.

Q 83. 30630558  Which of the following is not true as per the passage?

a)  Only ideas, titles, and names cannot be copyrighted.


b)  Copying factual ideas from a copyrighted work is an infringement.

c)  There can be no case of infringement for reproducing a work without consent after the expiry of the license.
d)  Indirect reproduction or copying is also an infringement.

Q 84. 30630558  X was a famous author who recently launched his new book, ' hum is desh ke vaasi hai.' The book is a fictional tale of a poor
person who is suppressed by affluent society members in many aspects. Y wrote a book after a couple of years, which depicted the struggle of a
middle-class worker who was laid off by his employer in an unfair manner. Now, X filed a suit for infringement, stating that the theme of Y's book is
the same as his book: the middle class or economically weaker section struggling because of the treatment by the affluent members of society. Pick
the most suitable option.

a)  There is an infringement as the main theme has been copied by Y.

b)  There is no infringement as the law prohibits exclusivity over ideas.


c)  There is no infringement, as fictional work cannot be protected.

d)  There is an infringement as this would negatively impact X's economic rights.

Q 85. 30630558  A, a prominent painter, has made a painting of a new and imaginary character which later on became a symbol of wisdom and
patience. To secure his economic rights, A got exclusive rights over his work. B, another painter, copied the character and added small details to
make it a bit fancier. Now, A became aware of this new version by B, which was made without his explicit consent. Hence, he wishes to sue B for
infringing upon his rights over the painting. Pick the most suitable option.

a)  There is an infringement since B did not share his earnings with A.


b)  There is no infringement since B has small details to make it fancier.

c)  There is an infringement since B has indirectly copied A's expression of an idea.

d)  There is no infringement since B characters are not copyrightable.

Directions for questions 66 to 105: Each set of questions in this section is based on the reasoning and arguments, or facts and principles set out
in the preceding passage. Some of these principles may not be true in the real or legal sense, yet you must conclusively assume that they are true
for the purposes of this Section. Please answer each question on the basis of what is stated or implied in the corresponding passage. Do not rely on
any principle of law other than the ones supplied to you, and do not assume any facts other than those supplied to you when answering the
questions. In some instances, more than one option may be the answer to the question; in such a case, please choose the option that most
accurately and comprehensively answers the question.
Disclaimer
All names and facts mentioned are hypothetical and are being used for educational purposes. Any resemblance with any person, institution or
situation is purely coincidental and unintentional.

Passage – 5

The concept of strict liability evolved from the case of Rylands v. Fletcher (1868). Strict liability means that anyone who retains dangerous chemicals
on their property is accountable for any errors made if those substances somehow escape and cause harm. If there was no negligence on the part of
the person retaining it, this rule is valid, and the burden of proof is always on the defendant to show why he is not guilty.

Following the precedent set by this case, even if a person did not act negligently when retaining a dangerous object on his property, he will be held
prima facie liable for any harm caused by that object's escape. A person is liable not because of their fault or negligence but because they kept a
dangerous object on their property, which then escaped and caused damage. The strict liability rule refers to the situation where liability arises even
in the absence of fault on the defendant's part.

Based on this principle, certain essentials have been created that help to decide whether liability is strict.
1. Someone must have brought something hazardous into their property.
2. There must be Non-natural use of land.
3. The hazardous item that was brought must escape and cause damage.

Exceptions of strict liability:


1. Plaintiff's own fault- Ponting vs Noakes would be a perfect example. In this instance, the plaintiff's horse entered the defendant's property, ate
some wild tree leaves, and passed away. The damage would not have been caused if the plaintiff's horse had not trespassed on the defendant's
property. Hence, the defendant was not held accountable. As there was no way out, the strict responsibility rule would not be applicable.
2. Act of God- Whatever natural occurrence that is unpredictable, uncontrollable, or unavoidable is not to be held responsible for any damage it
does.
3. Volenti non-fit injuria/ mutual benefit- If two people introduce something artificial for their mutual advantage and it causes damage, neither of them
may sue the other for compensation.
4. Act of stranger- The defendant will not be held accountable by this rule if any harm was brought by a third party over whom the defendant had no
influence.
5. Statutory authority- A defence to a tort claim is an act performed under the authority of a statute.

Source: https://blog.finology.in/Legal-news/strict-and-absolute-liability#:~: text=Both %20 systems %20of% 20law% 20are,


not%20permitted%20to%20raise%20defences
Q 86. 30630558  X was Y's coworker, whom Y had invited for dinner at his residence. As soon as he reached Y's house, he served X a cup of hot
soup, which was mistakenly spilled over him by Y's mistake. As a result, X sustained burn marks on his palm. Now, X wishes to sue Y under the
principle of strict liability. As per X, Y had an inherently dangerous thing that escaped due to his fault and caused damage. Pick the most suitable
option.

a)  
X was Y's coworker, whom Y had invited for dinner at his residence. As soon as he reached Y's house, he served X a cup of hot soup, which was
mistakenly spilled over him by Y's mistake. As a result, X sustained burn marks on his palm. Now, X wishes to sue Y under the principle of strict
liability. As per X, Y had an inherently dangerous thing that escaped due to his fault and caused damage. Pick the most suitable option.
b)  Y is not liable under strict liability, because X himself had agreed to come to his house.
c)  Y is liable under strict liability, because X sustained injuries due to his fault.

d)  Y is not liable under strict liability, because there was no escape of a dangerous thing.

Q 87. 30630558  X was a famous scientist who had built a secure research lab in his house. One of his shelves had a hazardous chemical which
was an integral part of his ongoing research. However, the bottle of that chemical was accidentally damaged by X's carelessness, leading to that
chemical being dispersed in the air. To control the situation, X immediately sprayed the antidote for that chemical, preventing any damage to people
in the vicinity. Later on, when everyone came to know about this incident, they wanted to sue X under the principle of strict liability. Pick the most
suitable option.
a)  X is liable, because a dangerous element escaped to the nearby area due to his fault.

b)  X is not liable, because he prevented any sort of damage to anyone.


c)  X is liable, because people got scared because of his carelessness.

d)  X is not liable, because he already had the antidote for that chemical.

Q 88. 30630558  A and B were neighbors, and they both agreed to have barbed wire around their houses and the common boundary to protect
against any robbery or theft. Now, B took up the responsibility of getting the best quality wires and installing them, as A was leaving for a week-long
vacation. Now, B thought of installing electric barbed wires to ensure greater protection without consulting A. When A returned, he was happy to see
the wires and thought of going closer for reviewing the quality of the wires. As soon as A touched the wires, he received a severe electric shock.
Now, he wishes to sue B for this act under strict liability, while B claims that A himself has agreed to get the wires, thereby bringing 'volenti non fit
injuria' in the case. Decide.

a)  B is liable since A was unaware of the nature of the wires.

b)  B can take the defense, as they both agreed to get the barbed wires. '
c)  B cannot take the defense; he should have asked A to get the electric wires.

d)  B can take the defense since touching barbed wires is anyways harmful.

Q 89. 30630558  X was a famous scientist who had built a secure research lab in his house. One of his shelves had a hazardous chemical which
was an integral part of his ongoing research. One day, a thief entered X's house and stole that fancy shelve. He went to a garden and started
breaking the shelve behind a tree. As soon as the chemical was released in the air, it caused damage to everyone who was in the vicinity. Now,
decide whether X should take the defense of an act of stranger or volenti non fit injuria, if people who were present near the thief wish to sue X?

a)  He should take the defense of volenti non fit injuria since the thief got affected by the chemical due to his own act.

b)  He should take the defense of an act of a stranger since he had no role in the escape of that chemical (which was caused by the act of that thief).
c)  He should take the defence of an act of a stranger, since the thief got affected by the chemical due to his own act.

d)  No defence shall be available for X.


Q 90. 30630558  M had a dangerous dog breed at his home, which was always kept in a cage. After a couple of years of keeping the dog, M was
sure that the cage was highly effective in restraining his violent movements. However, the dog broke the lock of that cage and went on biting a
passerby. It was found that the lock became rusty, so the dog could break it. Now, the passerby wishes to sue M under strict liability, while he claims
that rusting is a natural process, which qualifies as an act of god. Pick the most suitable option.

a)  Rusting is an act of god, and hence M is not liable.

b)  Rusting doesn't qualify as an act of god, so M is strictly liable.

c)  M is strictly liable since he kept a dangerous thing on his land.


d)  M is not strictly liable since he was not aware of the rusty lock.

Directions for questions 66 to 105: Each set of questions in this section is based on the reasoning and arguments, or facts and principles set out
in the preceding passage. Some of these principles may not be true in the real or legal sense, yet you must conclusively assume that they are true
for the purposes of this Section. Please answer each question on the basis of what is stated or implied in the corresponding passage. Do not rely on
any principle of law other than the ones supplied to you, and do not assume any facts other than those supplied to you when answering the
questions. In some instances, more than one option may be the answer to the question; in such a case, please choose the option that most
accurately and comprehensively answers the question.
Disclaimer
All names and facts mentioned are hypothetical and are being used for educational purposes. Any resemblance with any person, institution or
situation is purely coincidental and unintentional.

Passage – 6

In the law of torts, there is a duty on every person do acts with reasonable care in order to avoid any harm which may occur due to their failure of
taking such care.

This is the general rule in torts but there are certain exceptions which are allowed in these cases and these called as defences to tort. Under these
defences, a defendant can escape liability and volenti non-fit injuria is also one such defence which is available for the defendant.
In case a person gives his consent to doing of an act which leads to him getting injured, then even if an injury is caused by the other person, he
cannot claim any damages from that person because the act was one for which he voluntarily consented. The consent of the plaintiff acts as a
defence and this defence is called volenti non fit injuria which means to a willing person no injury happens.

For the application of the defence of volenti non fit injuria, there are some essential elements or conditions which should be present in a case and
only when they are fulfilled, this defence can be taken to prevent liability.

There are 2 essential elements in this defence:


• The plaintiff has the knowledge of the risk
• The plaintiff with the knowledge of risk has voluntarily agreed to suffer the harm.

Thus, whenever the plaintiff is aware of the possibility of harm which is likely to be caused by an act and when he still accepts to do that act and
therefore agrees to suffer the injury, a defendant is relieved of his liability.

But only having knowledge about the risk is not enough for the application of this defence, It is known as Scienti non fit injuria, which means that
mere knowledge does mean consent to the risk. Thus having knowledge is only a partial fulfillment of the conditions for the application of volenti non
fit injuria.

In the cases where the defendant is taking the defence of volenti non fit injuria, the burden of proof is on him to show that the plaintiff had full
knowledge of the act and he had consented to the risk involved in the act and the defendant has to show that the plaintiff was also aware of the
extent of risk which was involved in the act for successfully taking this defence.

Source: https://blog.ipleaders.in/volenti-non-fit-injuria/
Q 91. 30630558  M was admitted to XYZ hospital right after him suffering through a cardiac arrest. The doctor informed him that an open heart
surgery is the only way out of for him, since he has a chronic cardiac illness. However, the doctor failed to tell M about all the associated risks and
chances of developing some other ailments. M agreed for the surgery, that was later on performed successfully. However, he developed some
unusual ailment after a couple of months. After getting himself tested, it was found out that this is a side effect of an open heart surgery. M wishes to
sue the doctor, while the doctor claims that M himself consented to the surgery. Pick the most suitable option.
a)  The doctor will be liable because M did not have the knowledge about the risks.

b)  The doctor is not liable because he only wanted to save M from his chronic illness.
c)  The doctor is liable because he should have been more careful while performing the surgery.

d)  The doctor is not liabe because M himself consented to the surgery.

Q 92. 30630558  C and D went for a picnic on one of the weekends. There, both of them decided to have a round of archery, since they used to
participate in such tournaments during their school days. To try something new, they entered into a competition where both of them will put an apple
on their head (one by one) and the other person would shoot at it. B got hurt in this game as A missed upon hitting the apple. Now, B claims that he
didn't consent to the injury, and hence A is liable. Pick the most suitable option.

a)  A is not liable, because B himself consented to the game.

b)  A is liable, since B did not consent to the possibility of getting injured.
c)  A is not liable, since B should have not entered into such a dangerous competition.

d)  A is liable, because he missed upon hitting the apple.

Q 93. 30630558  A and B were neighbours. One day, B was leaving for his workplace when he was stopped by A. Asked B if he can drop him to the
nearby colony in his car because it was an emergency. B agreed to A's request. A sat in B's car, and B suddenly started speeding it up. A asked B to
slow down, but B did not pay any heed to this request. Due to the high speed, the car met with an accident. Decide if B is liable or he can take the
defense of volenti non fit injuria?

a)  B is liable since A did not agree to this act of speeding up the car.

b)  B can take the defense of volenti non fit injuria since A had agreed to sit in the car.
c)  B is liable, since he should have warned A that the car will move at a high speed.

d)  B can take the defense since road accidents are unpredictable in nature.

Q 94. 30630558  Had you been the author of this passage, which of the following statements would go against your concept of volenti non fit injuria?

a)  Knowledge alone is not enough for the defense of volenti non fir injuria.
b)  Scienti non fit injuria is a defnese where mere knowledge absolves the defendant from liability.

c)  It is on the defendant to prove that the plaintiff;s conduct fulfils all the prerequisites for volenti non fit injuria.
d)  Awareness about the extent of risk shall also be proved by the defendant while taking the defense of volenti non fit injuria.

Q 95. 30630558  X was supposed to undergo surgery for his damaged kidney. However, he argued with the doctor not to give him sedatives
because he wished to see the entire process. Hence, he was only given painkillers to remove the pain during the surgery. Just when the doctor was
making a cut in his stomach, X suffered a mental shock seeing so much blood. Now, he wishes to sue the doctor for not warning him that there might
be so much more blood than he could've anticipated. On the other hand, the doctor pleads for the defnse of 'volenti non fit injuria'. Pick the most
suitable option.

a)  The doctor can take the defense since X himself denied sedatives and was aware that there would be blood flowing out.

b)  The doctor cannot take the defense because he did not inform X about the exact amount of blood that might flow out.
c)  The doctor can take the defense because it was X who has argued with the doctor.

d)  The doctor cannot take the defense because he is responsible for the well being of the patients.

Directions for questions 66 to 105: Each set of questions in this section is based on the reasoning and arguments, or facts and principles set out
in the preceding passage. Some of these principles may not be true in the real or legal sense, yet you must conclusively assume that they are true
for the purposes of this Section. Please answer each question on the basis of what is stated or implied in the corresponding passage. Do not rely on
any principle of law other than the ones supplied to you, and do not assume any facts other than those supplied to you when answering the
questions. In some instances, more than one option may be the answer to the question; in such a case, please choose the option that most
accurately and comprehensively answers the question.
Disclaimer
All names and facts mentioned are hypothetical and are being used for educational purposes. Any resemblance with any person, institution or
situation is purely coincidental and unintentional.

Passage – 7
A minor is one who has not attained the age of 18, and for every contract, the majority is a condition precedent. By looking at the Indian law, minor's
agreement is a void one, meaning thereby that it has no value in the eye of the law, and it is null and void as it cannot be enforced by either party to
the contract. And even after he attains majority, the same agreement could not be ratified by him. The reason to this is a minor is incapable of giving
consent. The rule of estoppel also doesn't apply in case of minor as estoppel is a legal rule of evidence which prevents a party from alleging
something that contradicts what he previously stated. The court held that the doctrine of estoppel does not apply to the case in which the person
knows the real facts, before hand and here the attorney of the defendant knew that the plaintiff was a minor. Also, According to Section 64 of the
Indian Contract Act, when a person at whose option a contract is voidable rescinds it, the other party need not perform it. This applies to contracts
that are voidable, but a minor's contract is void, and therefore, he cannot be asked to refund the amount money to the moneylender. But there are
certain exceptions as well. Firstly, in a contract, a minor can be a promisee but not a promisor. So if the minor has performed his part of the promise,
but the other party hasn't the minor being in the position of a promisee he can enforce the contract. A contract entered into by guardian of minor for
his benefit a minor can sue the other party when it does not perform its promise. Under the Indian Apprentices Act, 1850, a contract of apprentice
entered by guardian on his behalf is binding on the minor. The apprenticeship contract is an employment contract between an employer and a young
person aged 16 to 29 who has completed his/her compulsory schooling (derogations are possible). And the last rule is if a person is incapable of
entering into a contract is supplied by another person with necessities of life, the person who has supplied is entitled to get reimbursement from the
property of such incompetent person, including a child as well. But if the minor has no property of his own, then he cannot be bound to reimburse the
other person.
Q 96. 30630558  To pay off, the promissory note and mortgage debt of his father, Shamu a minor and his mother, the Shamu sold a piece of land to
the holders of the promissory note in satisfaction of the debt. He paid off the mortgage and got possession of the land. But later the Shamu claimed
that because of his minority the contract was void, and he demanded the possession of land. Decide-

a)  The contract is a valid one as contract was entered by a Shamu on behalf of his father.

b)  The contract is a void one as contract with minors are void-ab-initio.

c)  The contract is valid one as contract was for the benefit of the minor and was entered into by his guardian; his mother.
d)  The contract is void one as a minor is not supposed to pay debt of his father.

Q 97. 30630558  Ram borrowed some money during his minority and after attaining the age of majority, he made a fresh promise to pay that sum
and interest. Decide-
a)  The contract was not enforceable due to the reason that consideration received during minority is not a good consideration.

b)  
The contract is enforceable as the promise to repay sum and interest are made during majority and thus now Ram being competent makes contract
enforceable.
c)  The contract is enforceable at option of another party.

d)  The contract is not enforceable as Ram is not mature.

Q 98. 30630558  S, while he was a minor received some goods from K in connection with his business and was indebted to him. When he attained
majority, he took some more money and executed a bond for paying the total amount to K. In an action by K to recover the said amount, it was
contended by S that he was not liable to pay as they purported to be in his minority. Decide

a)  S is liable to pay only that sum which he was supposed to pay in minority as it was for his benefit.

b)  S is not liable to pay any amount as he unemployed.

c)  S is liable to pay the whole amount since there was a new consideration attached.
d)  S is liable to pay only that amount which he took from K during time of majority.

Q 99. 30630558  Riya, when she was 15 years old lost both her parents in a car accident thus her family cook, Mahesh took Riya to his house and
took care of her. He admitted her to a good school payed for her education and even on receiving bonus at his job gifted her a car so she could drive
to school. Now Riya has attained majority and Mahesh wants all the expenses back and is asking Riya to start cooking with him for it. Decide-

a)  Mahesh took care of Riya like his own daughter, thus Riya should be grateful to him and start working.
b)  Riya should work and pay expenses only for necessities and not car and other unimportant stuff.

c)  Riya has no property of her own, thus she is not bound to reimburse Mahesh.

d)  All the work Mahesh did is not counted as necessity.

Q 100. 30630558  A, when he was a minor, mortgaged his property to B, a moneylender. At that time, B's attorney had the knowledge about A's age.
A later paid only Rs 8,000 but refused to pay rest of the money. A's mother was his next friend (legal guardian) at that time, so he commenced an
action against B saying that at the time of making of a contract, he was a minor, so the contract being a void one, he is not bound by the same.
Decide-

a)  A contract entered into by a minor is totally void. Thus A is not bound.

b)  A was having a legal guardian i.e. her mother, thus being his legal guardian she is liable for the same.
c)  Defendant had the knowledge about A's age. Thus, still entering into a contract is mistake on his part

d)  A by paying rs. 8000 made contract enforceable and now cannot plead defense of minority.

Directions for questions 66 to 105: Each set of questions in this section is based on the reasoning and arguments, or facts and principles set out
in the preceding passage. Some of these principles may not be true in the real or legal sense, yet you must conclusively assume that they are true
for the purposes of this Section. Please answer each question on the basis of what is stated or implied in the corresponding passage. Do not rely on
any principle of law other than the ones supplied to you, and do not assume any facts other than those supplied to you when answering the
questions. In some instances, more than one option may be the answer to the question; in such a case, please choose the option that most
accurately and comprehensively answers the question.
Disclaimer
All names and facts mentioned are hypothetical and are being used for educational purposes. Any resemblance with any person, institution or
situation is purely coincidental and unintentional.

Passage – 8

One of the essential requirements to form a contract is that it should not be void. Section 10 of the Indian Contract Act says, "all agreements are
contracts…that are not hereby expressly declared to be void". A contract can be void due to several reasons, for example:
1. It could be void ab initio- i.e., an agreement to do an illegal act is void ab initio, that is, it is void from the moment it was first made.
2. It is against the principles of justice and fairness or against public policy.
3. It becomes subsequently void because of a change in the law.
4. It has already been fully performed.

Some agreements are just harmful to society. They are against public policy. Some such agreements are Agreements in restraint of marriage, trade
or legal proceedings. These agreements are expressly declared to be void in the Indian Contract Act in Section 26, 27 and 28 respectively.

According to Section 26 of the Indian Contract Act, all agreements in restraint of marriage except that of a minor are void. Romans were the first to
delegitimize agreements that were in restraint of marriage. The basis of making agreements in restraint of marriage void is that marriage is a
sacrament and nothing should interfere in the institution of marriage, not even contracts. The idea behind this provision is to not snatch away the
personal right of every individual to marry someone of their own choice. It is important to note here that according to the section, agreements in
restraint of marriage of a minor are not void.
Q 101. 30630558  Nidhi who is a close friend of Rohit is in love with a guy named Kaalu. Rohit thinks Kaalu to be a bad suitor for Nidhi. So, in order
to stop Nidhi from marrying Kaalu makes an agreement with her that for sake of some consideration she will not marry Kaalu. Decide-

a)  The agreement is restraint on marriage as Kaalu is perfect suitor for Nidhi.

b)  The agreement is not restraint on marriage as Rohit is friend of Nidhi and he is doing this all for good of Nidhi.

c)  The agreement is not restrain on marriage as Nidhi is a Minor.

d)  This agreement is one in restraint of marriage and is thereby void.

Q 102. 30630558  Rahul and Anjali loves each other very much. One day while sitting in a park Anjali says to Rahul that she is doubtful about Rahul
marrying her. So in order to win her trust Rahul contended that if he marries any other person except the her, he would give her 1000 pounds within
three months of his marriage. Decide-

a)  There is no promise to be carried out on by either of the party and promise is explicitly restrictive in nature. Thus, agreement is void.
b)  Boys cheat very often. So it is right on part of Rahul to make such a promise to Anjali.

c)  This is a void agreement as all the agreement on restrain of marriage are void.

d)  1000 Pounds is not a good consideration. Thus, agreement is void.

Q 103. 30630558  Romeo and Juliet loved each other but they were from different caste. Juliet promised to marry Romeo someday. After few weeks,
Juliet's father came to know about affair of Juliet with Romeo and very next day organized marriage of Juliet with some William. But Juliet also
happily agreed as William was her childhood crush. Romeo filed a suit asking the court for an injunction on the Juliet's marriage to the other person.
Decide
a)  Juliet should get punished for breaking Romeo's heart.

b)  Court shall dismiss Romeo's petition as stopping marriage of Juliet with person of her choice would be restrain on marriage.
c)  Court shall dismiss Romeo's petition as granting injunction is not function of court.

d)  Court shall entertain Romeo's petition as Juliet promised to marry Romeo and now she cannot breach the contract.

Q 104. 30630558  X, a legal guardian of A stops her from marrying till she attains 18 years of age and A agrees to this. Decide Whether it can be
challenged in court as restraint on marriage.

a)  This agreement can be challenged in court. b)  This agreement cannot be challenged in court.
c)  This agreement can be challenged when A attains majority. d)  None of the above.

Q 105. 30630558  A says "I will give you 10 lakh rupees if you do not marry your entire life." S accepts this offer but after two years gets married. A
after knowing about S's marriage files a suit against S. Decide-

a)  A will not succeed as any agreement which stops a person from getting married is null and void as per law

b)  A will succeed as S has breached the contract.

c)  S needs to give divorce to his spouse.


d)  A will not succeed as 'not marrying' is no consideration and without consideration contract cannot come into existence.

Logical Reasoning
Direction for questions 106 to 135: Read the following passages and answer the questions that follow.

Passage – 1

Can countries be sued under international law for failing to avert climate emergencies? On Wednesday, the UN General Assembly (UNGA) asked
the International Court of Justice to weigh in on the issue. That the resolution, sponsored by the small Pacific Island nation, Vanuatu, was adopted
unanimously by the UNGA testifies to the global consensus on the climate crisis. But it also reflects the frustration of the international community -
especially of small island countries, many of whom face an existential threat - with the procedures of the global climate agencies, particularly the
UNFCCC. Their deliberations often end up in compromises that delay climate action. The Hague-based court's opinion will not be binding but its
pronouncements carry moral weight. The advisory could set the stage for countries incorporating climate justice in their legal frameworks - akin, for
instance, to the way the UN Declaration of Human Rights has found resonance in statute books across the world. As the resolution pointed out, "it
will provide clarity to states on their obligations under international law to protect their people, now and in the future, from climate impacts".

This is not the first time that climate change has been taken up at a non-environmental UN forum. Global warming has been part of the UN Security
Council's agenda since 2007. From time to time in the past 15 years, the UNSC has tried to frame the issue from a security standpoint, instead of
looking at it from only a developmental or environmental perspective. But developing countries, including India and China, have rightly resisted the
securitisation of climate change. Its use of the vocabulary of rights and justice has given the Vanuatu-sponsored proposal more traction. The
initiative's success should also be seen in the backdrop of countries asserting their right to reparations after climate emergencies - Pakistan after last
year's floods, for instance.
Holding individual countries or governments to account for their climate inaction will, however, pose challenges. The issue has been a major
stumbling block at several climate meets. The Paris Agreement has a clause specifying that the pact "does not involve or provide a basis for any
liability or compensation" - it was inserted under pressure from US diplomats. American support for the UNGA resolution was, reportedly, a reluctant
one.

The UNGA's intervention should not detract from the task of reforming the UNFCCC. The institutions of the umbrella climate agency need to be
more equity-sensitive and justice-oriented. Engaging with the ICJ could push it towards this direction. The UNFCCC will, however, require much
more initiative from its wealthier members.
Q 106. 30630558  What can be inferred to be the main reason for the UNGA's resolution to ask the ICJ to weigh in on whether countries can be
sued under international law for failing to avert climate emergencies?

a)  To hold individual countries accountable for their climate inaction


b)  To securitize climate change from a global security standpoint

c)  To reform the UNFCCC to be more equity-sensitive and justice-oriented


d)  To provide clarity to states on their obligations under international law

Q 107. 30630558  What is the main conclusion of the passage?

a)  
The UNGA's call for the ICJ to weigh in on whether countries can be sued under international law for failing to avert climate emergencies could lead
to countries incorporating climate justice in their legal frameworks.
b)  
The UN Security Council's attempts to frame climate change as a security issue have been resisted by developing countries like India and China
because these countries need to ensure the development of their people first.
c)  
The institutions of the UNFCCC need to be more equity-sensitive and justice-oriented along with ensuring that the ideals of justice and equality are
upheld globally, and engaging with the ICJ could push them towards this direction.
d)  
The Paris Agreement's clause specifying that the pact "does not involve or provide a basis for any liability or compensation" was inserted under
pressure from the US which is aligned with its reluctance to accept accountability.

Q 108. 30630558  Which of the following best describes the role played by the second paragraph in the passage?

a)  
The paragraph highlights the challenges inherent in holding individual countries or governments to account for their inability or inaction when it
comes to addressing matters of climate change and emergencies.
b)  
The paragraph discusses the clause in the Paris Agreement that specifies that the agreement does not provide a basis for any liability or
compensation and how it has rendered all subsequent discussions on the issue ineffective.
c)  
The paragraph provides background information on previous attempts to address climate change at the UN and highlights the importance of the
Vanuatu-sponsored proposal's approach in gaining global consensus on the issue.
d)  
The paragraph contrasts the language of the Paris Agreement and the current proposal and argues that the use of the vocabulary of rights and
justice has provided the Vanuatu-sponsored proposal greater traction and validity.

Q 109. 30630558  Which of the following external evidence would most weaken the author's argument that the ICJ's advisory on suing countries for
failing to avert climate emergencies could push the UNFCCC towards being more equity-sensitive and justice-oriented?

a)  
A study by a prominent climate change agency that provides definitive evidence that individual countries have little impact on global carbon
emissions.
b)  
A study which proves that the UNFCCC is governed completely by the individual agendas of its wealthier member nations who only wish to promote
their development.
c)  A news article about a recent ruling by a national court that dismissed a case against the government for failing to take action on climate change.
d)  
A survey of public opinion in several wealthy countries that shows low levels of support for compensating developing countries for the costs of
climate change.

Q 110. 30630558  Which of the following statements could be a valid inference based on the information provided in the passage?

a)  
Small island countries like Vanuatu face an existential crisis due to the biased decisions of agencies like UNFCCC that are neither equitable or just.
b)  
The UNGA would like to facilitate the reformation of climate change agencies like UNFCCC but its wealthier member nations actively resist the idea.
c)  There have been instances of countries seeking reparations, which refers to compensation for damages, for climate emergencies faced by them.

d)  The ICJ's opinions on most matters that are not under its direct purview are always in the form of advisories that carry no legal implications.

Q 111. 30630558  Which sentence in the passage explains the reason for the Paris Agreement's clause on liability and compensation?
a)  "The UNFCCC will require much more initiative from its wealthier members."

b)  "The Hague-based court's opinion will not be binding but its pronouncements carry moral weight."
c)  "The initiative's success should also be seen in the backdrop of countries asserting their right to reparations after climate emergencies."

d)  
"The Paris Agreement has a clause specifying that the pact 'does not involve or provide a basis for any liability or compensation' - it was inserted
under pressure from US diplomats."

Direction for questions 106 to 135: Read the following passages and answer the questions that follow.

Passage – 2

Garages, automobile workshops and service stations engage children in violation of the law, a report by India's apex child rights organisation says.

"Child labour in automobile industry", released by the National Commission for the Protection of Child Rights (NCPCR), has held the big automobile
manufacturers equally responsible with the garages.

The Child Labour (Prohibition & Regulation) Act 2016 debars any organisation from engaging children below 14 years in any occupation or process.
It prohibits the employment of adolescents - those aged 14 to 18 -- in hazardous occupations or processes, including automobile workshops.

The NCPCR report is based on video surveys, containing interviews with child labourers from the automobile sector in Guwahati, Indore, Meerut,
Chennai and Patna. The surveyors found children earning between Rs 2,000 and Rs 2,500 a month by working in various motor garages in the cities
surveyed.

The automobile market has given rise to a huge basket of allied businesses such as repairs, maintenance and spare parts. The car and bike makers
do not open service centres in proportion to the number of vehicles sold, nor do they manufacture spare parts in adequate numbers, the report says.

Local mechanic shops and garages have thus mushroomed, and this has led to the emergence of a grey market, it adds.
However, the survey has a couple of weaknesses --- it is anecdotal and does not provide rigorous and adequate data that can lend itself to statistical
sampling. Nor does it provide recommendations, putting the "onus" instead on the auto industry to stop the garages from engaging child labour.

"The big automobile companies love the Indian auto market, but when it comes to obeying laws, these companies shy away from taking
responsibility," the report says.

NCPCR chairperson Priyank Kanoongo said the survey's objective was to sensitise the industry into taking responsibility.

"It was a video survey to give an idea about the problem. Our objective is that all the units from the manufacturers to the service stations should
come under the organised sector and follow labour standards. This will stop child labour," he said.

Shailabh expressed dismay at the poor allocation of funds for the National Child Labour Programme (NCLP), the central government's principal
instrument to rescue and rehabilitate child labourers. The budget allocation for the NCLP plummeted from Rs 33 crore in 2022-23 to Rs 20 crore in
2023-24 - a fall of nearly 40 per cent.

"According to the 2011 census, there were 10 million child labourers in the country," Shailabhsaid.

"There is no platform to know how many cases have been registered against employers, and how many have been prosecuted and convicted." Dev
Nathan, labour economist and a professor at the Institute of Human Development, Delhi, said the law against child labour has two big loopholes.

One, it allows children to be engaged in family-based enterprises after school hours and two, it lacks any provision to hold the lead firm responsible
for anti-labour practices relating to outsourced work done outside its premises. "The lead firms should be held accountable for the anti-labour
practices followed by different service providers and spare part manufacturers. In addition, the permission to work in a family enterprise is a potential
facilitator for child labour and should be dropped," he said. These demands have been raised in the past but the response from the government has
been negative, he said.
Q 112. 30630558  According to the National Commission for the Protection of Child Rights (NCPCR) report, what is the primary factor contributing to
the prevalence of child labor in the automobile industry in India?
a)  Inadequate manufacturing of spare parts by automobile manufacturers.

b)  Insufficient budget allocation for the National Child Labour Programme (NCLP).

c)  Lack of strict enforcement of the Child Labour (Prohibition & Regulation) Act 2016.

d)  Reluctance of big automobile companies to take responsibility for their actions.

Q 113. 30630558  "Shailabh expressed dismay at the poor allocation of funds for the National Child Labour Programme (NCLP), the central
government's principal instrument to rescue and rehabilitate child labourers. The budget allocation for the NCLP plummeted from Rs 33 crore in
2022-23 to Rs 20 crore in 2023-24 - a fall of nearly 40 per cent." Based on the given information, which of the following statements can be inferred?

a)  The number of child laborers in India has significantly decreased from 2022-23 to 2023-24.

b)  The government's commitment to combating child labor has weakened over the years.

c)  The National Child Labour Programme (NCLP) has been successful in its efforts to rescue and rehabilitate child laborers.
d)  The budget allocated for the National Child Labour Programme (NCLP) is directly proportional to the prevalence of child labor in India.

Q 114. 30630558  Which of the following, if true, would most strengthen the argument that the decrease in budget allocation for the National Child
Labour Programme (NCLP) indicates a weakened commitment from the government to combat child labor?

a)  The prevalence of child labor in India has remained consistent or increased during the period of reduced budget allocation.

b)  Other government programs aimed at addressing child labor have also experienced a decrease in budget allocation.
c)  Civil society organizations have reported an increase in the number of child labor cases during the period of reduced budget allocation.

d)  The government has implemented stricter enforcement measures and penalties against organizations found to be engaging in child labor.

Q 115. 30630558  Which of the following is an assumption made in the passage?

a)  The automobile industry in India is the primary sector responsible for engaging children in violation of child labor laws.

b)  
The video surveys conducted by the National Commission for the Protection of Child Rights (NCPCR) provide accurate and representative data on
child labor in the automobile sector.
c)  The reluctance of big automobile companies to take responsibility is the sole factor contributing to the prevalence of child labor in the industry.

d)  
The decrease in budget allocation for the National Child Labour Programme (NCLP) directly correlates with a decrease in the number of child
laborers in India.

Q 116. 30630558  Based on the information in the last paragraph of the passage, which of the following can be inferred?

a)  
The lead firms in the automobile industry are currently held accountable for anti-labor practices relating to outsourced work done outside their
premises.

b)  
The government has been responsive to the demands raised by labor economists and has taken measures to eliminate the permission for children
to work in family enterprises.
c)  
The government has previously responded positively to demands for holding lead firms accountable for anti-labor practices, but the current response
has been negative.
d)  
The labor economist mentioned in the passage has been successful in persuading the government to drop the permission for children to work in
family enterprises.

Q 117. 30630558  Based on the information in the passage, which of the following represents the most logical course of action to address the issue
of child labor in the automobile industry?

a)  Increase the budget allocation for the National Child Labour Programme (NCLP) to its previous level or higher.
b)  Implement stricter enforcement measures and penalties against both big automobile manufacturers and local garages engaging in child labor.

c)  Encourage big automobile companies to open more service centers and manufacture spare parts in adequate numbers.
d)  Conduct comprehensive research studies to collect rigorous and adequate data on child labor in the automobile sector.

Direction for questions 106 to 135: Read the following passages and answer the questions that follow.

Passage – 3

It is not that Ranajit Guha's death was unexpected. He was in his hundredth year and had been unwell for some time. But, death, in spite of its
inevitability, leaves us invariably with a sense of unpreparedness. Thus, it was: a sense of profound loss that robbed me of sleep when the news of
his end came around midnight on 28 April. A permanent settlement if ever there was one. An ineradicable line drawn.

Ranajit Guha had entered my intellectual landscape at least a decade before I first met him. My intellectual introduction to him was through his first
book, A Rule of Property for Bengal.

There are some first impressions that remain. The Preface was written in the third person. The subtitle of the book, "An Essay on the Idea of
Permanent Settlement'' puzzled me. I knew of the Permanent Settlement as an important, if pivotal, chapter of Bengal's agrarian history. Agrarian
history brought to my mind books like those written by Irfan Habib or N.K. Sinha. What had ideas got to do with agrarian history? I started reading
the book with a sense of awe, puzzlement and trepidation. I was bowled over by it not because I understood everything in it let alone comprehending
the significance of its arguments. I was overwhelmed by how well the book was written, by the sheer beauty and euphony of Ranajit Guha's prose.
So many of Guha's sentences and turns of phrase became embedded in my memory. Could history really be written with such style and grace, I
asked myself. I continue to believe that stylistically it is the best written book on any aspect of Indian history. Through subsequent readings, I also
came to recognize its historiographical originality and significance.

When the book was conceptualized and written (first an incomplete version in Bengali in the mid-1950s and then in English and published by Mouton
and Company in 1963), historians did not consider ideas to be a factor in the making of British policy in the colonies. Policies were made through
pressures of factions formed by self-interested individuals. Guha's book broke the shackles of this mindless history. He showed how ideas like
mercantilism, free trade and physiocracy had influenced the formulation of a major agrarian policy in late 18th century Bengal. He argued that the
priorities of colonialism - the quest for expanding markets - had divorced the original idea of the policy from its outcome. The zamindar who was
supposed to have become through the magic touch of property an improving capitalist farmer had turned out to be its exact opposite - a parasitic
absentee landlord.
Q 118. 30630558  Which of the following is the author most likely to agree with?

a)  Ranajit Guha was the first historian who left an impression on the author's mind regarding the Permanent Settlement.
b)  Ranajit Guha introduced the importance of ideas as a factor in Britain establishing a colony in Bengal.

c)  Historians seduce their readers with the use of flowery prose.

d)  Ranajit Guha's take on agrarian history was not novel compared with N.K Sinha.

Q 119. 30630558  Based on the author's arguments, which of the following, if true, will establish Guha's take on certain aspects of history as novel?

a)  Guha introduced new way of looking at history through a form hitherto unpractised by historians.

b)  Guha brought into view that Bengals' agrarian history can be understood through permanent settlement.
c)  Guha showed the world that the British colonisers were motivated by greed.

d)  Guha took on historians who mindlessly recorded facts without attesting analysis to them.

Q 120. 30630558  Which of the following, if true, would most weaken the author's arguments?

a)  Guha did not comment on the protests against the zamindars that were erupting across Bengal in that era.

b)  The author, admittedly as student back then, was underexposed in the vastness of then contemporary history writing.
c)  
The author did not look at the works of other historians who had been writing on and recording the problems of agrarian Bengal under the British
colonisers.
d)  Guha looked at the problems of expanding markets from the perspective of colonialism and not capitalism.

Q 121. 30630558  Based on the author's arguments, which of the following must necessarily be true?

a)  The zamindars were once farmers.


b)  Government policies are made to protect the interest of the upper echelons of society.

c)  Agricultural is steeped in the history of ideas.


d)  Property consolidates its holders with unbridled confidence.

Q 122. 30630558  Which of the following is the author most likely to agree with?

a)  Guha comes from a lineage of historians who put emphasis on analysis as well as good writing.
b)  Agrarian policies are charted keeping free trade in mind.

c)  Both (a) and (b).


d)  Neither (a) nor (b).

Q 123. 30630558  Which of the following can be inferred from the given passage?

a)  The zamindars under colonialism was an extension of the ideas which governed the colonial enterprise.
b)  The quality of a book being well written is always overwhelming to the readers.

c)  Bengal's history of agriculture needs to be looked at only from the perspective of Permanent Settlement.

d)  Capitalism allows farmers to own properties.

Direction for questions 106 to 135: Read the following passages and answer the questions that follow.

Passage – 4

Literature is a genre of art that strives to exhibit originality and inventiveness via the written word. In this regard, literature functions in the same
manner as any sort of music. To get into more detail on the topic, it is necessary to have an understanding of the term "art." The simplest way to
describe art is as any kind of communication that takes place between the person who generates the material by using their imagination and the
audience that is on the receiving end of the production. Every piece of writing an author creates has a narrative that the writer is working to develop
and share with their audience in some way. These narrative paints a picture of the characters in question for the reader. Additionally, the readers are
able to go to new locations and even discover significance in what at first glance may seem to be unremarkable occurrences. The manner of writing
that an author employ is, in essence, a medium for the dissemination of art. It's possible that the lyrics of some poets may be seen as a meaningful
and dynamic system at the same time that's defined by ontological completeness. On the other hand, the composition of a lyric is not a fixed process
but rather an ongoing one that evolves over time. The combination between a poet's view of the world, his reasoning ability, and his imagination
results in the formation of the poet's lyrical cosmos. Not only does the author represent the objective reality via the process of writing songs, but he
also communicates his understanding of the world and provides an evaluation of it.
Q 124. 30630558  In the light of the passage, it can be understood that:

a)  An author writes in order to monetize art forms. b)  An author writes in order to spread art forms.

c)  An author writes in order to segregate art forms. d)  An author writes in order to negate art forms.

Q 125. 30630558  Out of the following options, which one is factually correct in the light of the passage?

a)  The author considers his audience to be unimportant.

b)  The author feels that his audience should buy his books.
c)  The author considers his audience to be important.

d)  The author considers the gender of his audience to be important.

Q 126. 30630558  According to the passage, the author's views are:

a)  an appraisal of the world. b)  a denial of the world.

c)  an exaggeration of the world. d)  a satire on the ways of the world.

Q 127. 30630558  One of the author's tasks is to:

a)  promote cultural amalgamation. b)  represent the unreal elements of the world.

c)  represent the subjective reality. d)  represent the objective reality.

Q 128. 30630558  What does the author mean when he says that the composition of lyric evolves over time?
a)  The composition of lyric depends upon the social support that the author receives.

b)  The composition of lyric is an arduous process.


c)  The composition of lyric develops over time.

d)  The composition of lyric cannot be interpreted fully.

Q 129. 30630558  How has art been defined in the passage?

a)  Communication between a person who draws inspiration from great works and his readers.

b)  Communication between a person who always tries to project elevated ideas and his reader who wishes to gather more knowledge.
c)  Communication between a person who aims to reform the society and the individuals who reside in the society.

d)  Communication between a person who imagines, creates and the audience that is at the receiving end.

Direction for questions 106 to 135: Read the following passages and answer the questions that follow.

Passage – 5

What provokes the act of Bragging? Actually, this is the question which is easier to ask than answer. Pursuant to the article "Science reveals why we
brag so much", process of talking about ourselves, whether in a personal conversation or through social media, triggers the same sensation of
pleasure in the brain as food or money. About 40% of everyday speech is devoted to telling others about what we feel or think. Particularly, the act of
bragging was accompanied by spurts of heightened activity in brain regions belonging to the meso-limbic dopamine system, which is usually
associated with the sense of reward and satisfaction from food or money.

In addition, we can add several more factors which provoke the act of bragging, they are: inferiority complex, narcissism, egocentrism, spiritual
hunger and lack of genuine, real-life relationships, willingness for selfdemonstration, desire to compete with others, longing for audience
manipulation in order to gain public recognition and esteem, especially if you are a public figure such as a politician. As for Bragging Types, based on
the prospective of our study, we will mainly focus on humble bragging, which is very subtle, discreet and heavily camouflaged form of self-promotion.
Another common type of bragging which is widespread in political discourse and stands in opposition with humblebragging is called direct type of
bragging. Unlike humblebragging, it is very ostensible, vivid and does not require additional efforts to read between lines. It is common for politicians
to brag about their party, accomplishments and success they have achieved or brag about their connections with some late public figures, which
were once widely prominent. Sometimes, direct type of bragging becomes very hyperbolized and might evoke aggression among listeners.
Q 130. 30630558  Out of the following options, which one is a possible reason of bragging?

a)  Ignorance b)  Self-love c)  Racism d)  Dishonesty

Q 131. 30630558  What is the author's opinion about politicians?

a)  Politicians are dishonest people who have no ambition in life.

b)  Politicians work very hard to achieve something in life.


c)  The politicians try to hide their attainments.

d)  The politicians take pride in their attainments.

Q 132. 30630558  In the light of the passage, it can be inferred that:

a)  Bragging can take the form of exaggeration. b)  Bragging can take the form of personification.

c)  Bragging can take the form of an oxymoron. d)  Bragging can take the form of a subtle satire.

Q 133. 30630558  What does the author mean to say when he mentions the phrase 'reading between the lines'?

a)  look for a meaning that is implied. b)  look for a meaning that is explicit.

c)  look for a meaning that is most important. d)  look for a meaning that is socially relevant.

Q 134. 30630558  The first paragraph of the passage can be defined as:

a)  subjective. b)  a diatribe. c)  an empirical kind. d)  a research methodology.

Q 135. 30630558  Bragging can be characterized as:

a)  dilatory in nature. b)  provocative in nature. c)  communal in nature. d)  suicidal in nature.


Mathematics
Directions for questions 136 to 140: Answer the questions on the basis of the information given below.

A survey was conducted on the monthly average earnings of four different categories of people - A, B, C and D. The average monthly income of the
self-employed is Rs. 15,000 for Category A and for Category B is Rs. 6,500. The average monthly income in salaried employment for Category A is
Rs. 21,000 and for Category B is Rs. 16,800. Moreover, the average monthly income in casual employment for Category A is Rs. 10,000 and for
Category B is Rs. 5,500.

The average monthly income in salaried employment for Category C is Rs. 20,300 and for Category D is Rs. 15,500. The average monthly income of
the self-employed is Rs 15,300 for Category C and for Category D is Rs. 11,000. The average monthly earning of casual employment is Rs.8,500 for
category C and for category D is Rs. 7,500.
Q 136. 30630558  Which of the following statements is FALSE based on the information given in the paragraph?

a)  The ratio of the average monthly income of salaried individuals from Category A and Category B is 5 : 4 respectively.

b)  The average monthly income of self employed individuals from Category A is three times that of individuals in casual work from Category B.

c)  The average monthly income of self employed individuals from Category C is 80% more than those in casual work of the same category.
d)  The average monthly income of individuals from Category B in casual work is half that of self employed individuals from Category D.

Q 137. 30630558  Among the salaried individuals, the average monthly income of Category C is what percentage more or less than that of Category
B?

a)  Between 8% and 12% b)  Between 15% and 20% c)  Between 20% and 25% d)  Between 25% and 28%

Q 138. 30630558  An individual in casual work from Category C deposited his total annual income in a scheme that gave him 13% simple interest
per annum and a self employed individual from Category B invested his total annual income in a Mutual Fund. If after 2 years both of them got the
same interest, then what is the simple annual rate of interest for the Mutual fund?
a)  15% b)  18% c)  21% d)  17%

Q 139. 30630558  If the average monthly income of the self-employed person across all categories A, B, C and D respectively increased in the ratio
of 2 : 4 : 3 : 5 such that the difference between the average monthly income of selfemployed person of category C and B is Rs. 7,200, then what is
the difference between the average annual income of self-employed person of category D and A?

a)  Rs. 9,600 b)  Rs. 10,200 c)  Rs. 9,200 d)  Rs. 10,640

Q 140. 30630558  Three individuals - one self-employed, one salaried and one in casual work - from Category A start a business venture with their
total annual incomes. Each one invests for a year and at the end of the year the profit from the business was Rs.2,25,400. What was the share in
profit of the salaried individual?

a)  Rs.1,44,200 b)  Rs.1,02,900 c)  Rs.1,01,200 d)  Rs.1,13,500

Directions for questions 141 to 145: Answer the questions on the basis of the information given below.

The following pie charts show the number of students enrolled in various courses - Accountancy (ACC), Business (BUS), Economy (ECO), English
(ENG), History (HIS), Maths (MAT), and Psychology (PSY) in a university for the Spring and Fall semesters. In spring semester total of 600 students
enrolled and in fall semester total of 650 students enrolled.
Q 141. 30630558  In how many courses enrolment in Spring semester is less than the enrolment in fall semester?

a)  7 b)  6 c)  5 d)  4

Q 142. 30630558  Which course had the largest percentage increase in enrolment from the Spring to the Fall semester?

a)  HIS b)  PSY c)  MAT d)  BUS

Q 143. 30630558  If the average enrolment in all courses for the Fall semester was the same, then minimum how many more students should be
enrolled in Fall semester?

a)  0 b)  1 c)  8 d)  15

Q 144. 30630558  If the university wants to increase the enrollments in ECO by 50%, ENG by 25%, MAT by 20% and PSY by 25% and rest
remaining the same, in the Spring semester, then what is the percentage increase in the enrollment with respect to the current enrollment?

a)  21% b)  34% c)  19% d)  17%


Q 145. 30630558  What is the ratio of the total enrollments in ENG during Spring and Fall semesters respectively?

a)  12 : 7 b)  13 : 6 c)  18 : 13 d)  18 : 17

Directions for questions 146 to 150: Answer the questions on the basis of the information given below.

An appliance store sells four different articles - A, B, C and D. The selling price and cost price of article A are 2x2 - 25x and x2 + 15x respectively and
the profit earned by the sale of article A is Rs.4,500. The selling price of article B is x2 + 75x whereas the profit earned by its sale is 25x. Article C
has cost price of 40x and a profit of 14x. The cost price of article D is x3/50 and it is marked 25% above cost price.
Q 146. 30630558  What was the cost price of article A?

a)  Rs.12,450 b)  Rs.9,450 c)  Rs.8,950 d)  Rs.10,250

Q 147. 30630558  If article B was marked 20% above cost price, then find the amount given as discount.

a)  Rs.270 b)  Rs.450 c)  Rs.280 d)  Rs.190

Q 148. 30630558  If there is a discount of 8% on article D, then what is the selling price of the same?

a)  Rs.20,658 b)  Rs.15,455 c)  Rs.16,767 d)  Rs.15,957

Q 149. 30630558  What percentage above the cost price was article C marked up if the discount offered was Rs.1,620?

a)  20% b)  80% c)  60% d)  40%

Q 150. 30630558  What is the absolute difference between the cost price of articles A and D?

a)  Rs.4,500 b)  Rs.5,130 c)  Rs.4,800 d)  Rs.5,200

You might also like